Peds

Lakukan tugas rumah & ujian kamu dengan baik sekarang menggunakan Quizwiz!

Mr. and Mrs. Foster have come to a genetics clinic because she is 40 years old and pregnant for the first time. They seek advice regarding Down syndrome (DS). Provide answers to the following questions about Down syndrome that the Fosters ask. (4 of 4 questions) What is the risk of having a child with DS when a mother is 40?

1 in 106 Rationale: This statistic is the relationship between the maternal age at the expected time of delivery and the estimated risk of DS.

Case #1: Cerebral Palsy (1 of 3) Tony is a 2-year-old boy who was born 8 weeks prematurely. He has been diagnosed with spastic cerebral palsy (CP). Identify five clinical signs or symptoms that are seen in children with spastic CP.

1. Increased muscle tone 2. Increased deep tendon reflexes and clonus (sudden dorsiflexion of the ankle or rapid distal movement of the patella resulting in alternating spasm and relaxation of the muscles being stretched) 3. Flexor, adductor, and internal rotator muscles more involved than extensor, abductor, and external rotator muscles 4. Difficulty with fine and gross motor skills 5. Most common contracture is that of the heel cord. 6. Hip adductor contractures leading to progressive subluxation and dislocation 7. Knee contractures 8. Scoliosis common 9. Typical gait is crouched, in-toeing, and scissoring. 10. Elbow, wrist, and fingers in flexed position with thumb adducted 11. Motor weakness of antagonist muscle groups

Mr. and Mrs. Foster have come to a genetics clinic because she is 40 years old and pregnant for the first time. They seek advice regarding Down syndrome (DS). Provide answers to the following questions about Down syndrome that the Fosters ask. (1 of 4 questions) We understand there are different types of DS. What is the most frequent type?

92% to 95% of cases are due to an extra chromosome 21, or trisomy 21 Rationale: Although the etiology is unknown, the cytogenetics of the disorder is well established. Additional forms of DS include translocation of chromosome 21 and mosaicism, which refers to cells with both normal and abnormal chromosomes.

Case 1: Cystic Fibrosis (4 of 5) David is the 3-year-old son of parents who are 38 and 40 years old. Mr. and Mrs. Bradley have been married for 15 years and underwent fertility therapy to have a child. David has a history of frequent colds increasing in severity during the past year. In addition, David has been losing weight, although he eats well. Mrs. Bradley has noticed during the past year that David has been having large bulky, foul-smelling bowel movements. David is admitted to the pediatric unit for a workup for cystic fibrosis (CF). Discuss the effects of increased viscosity of mucous gland secretions on the following: A. Bronchi B. Small intestine C. Pancreatic ducts D. Bile ducts

A. Chronic pneumonia and emphysema B. Meconium ileus C. Malabsorption syndrome D. Portal hypertension Rationale: Increased mucous gland secretion and mucus viscosity result in mechanical obstruction, which leads to chronic complications of the bronchi, small intestine, pancreatic ducts, and bile ducts.

Case 2: (4 of 5) Bill, age 8 years, is brought to the clinic because of acute respiratory distress. He appears anxious and is clinging to his father. The diagnosis is asthma. Instructions for using the MDI include to A. breathe medication in slowly. B. avoid breathing too deeply. C. hold inhaler at about a 45-degree angle to the mouth. D. breathe in through nose and out through mouth

A. breathe medication in slowly Rationale: A. A slow, deep inspiration held for 5 to 10 seconds will allow the medication to reach the narrow, deep airways. Rapid inspirations cause the medication to move through the unobstructed bronchioles to patent airways, where they are less needed. B. A deep breath is desired to allow the medication to reach the narrowed airways. C. The inhaler should be held upright with the mouthpiece in the mouth. D. Inspiration should be through the mouth to draw the medication into the lungs, and expiration should be through the nose. breathe medication in slowly

Case 1: (4 of 4) Kimberly, age 9 years, is brought to the clinic because of acute respiratory distress. She appears anxious and is clinging to her father. The diagnosis is asthma. Kimberlys father asks the nurse whether she can still participate in sports. The nurses response should be based on an understanding that A. exercise should be encouraged. B. exercise should be discouraged. C. organized sports are too strenuous for children with asthma. D. quiet activities such as reading are best for children with asthma.

A. exercise should be encouraged Rationale: It has been found that moderate or even strenuous exercise is advantageous for children with asthma, provided the asthma is under control. Restrictions on exercise are invoked only when the childs condition makes it necessary. Prophylactic treatment with ß-adrenergic agents or cromolyn sodium before exercise will usually permit full participation in strenuous exercise. The child should be encouraged to live a normal active life, including exercise. Premedication before exercise can permit full participation in strenuous exercise.

Case 2: (5 of 5) Bill, age 8 years, is brought to the clinic because of acute respiratory distress. He appears anxious and is clinging to his father. The diagnosis is asthma. Bill's father asks the nurse whether he can still participate in sports. The nurses response should be based on an understanding that A. exercise should be encouraged. B. exercise should be discouraged. C. organized sports are too strenuous for children with asthma. D. quiet activities such as reading are best for children with asthma

A. exercise should be encouraged. Rationale: It has been found that moderate or even strenuous exercise is advantageous for children with asthma, provided the asthma is under control. Restrictions on exercise are invoked only when the childs condition makes it necessary. Prophylactic treatment with ß-adrenergic agents or cromolyn sodium before exercise will usually permit full participation in strenuous exercise. The child should be encouraged to live a normal active life, including exercise. Premedication before exercise can permit full participation in strenuous exercise.

Case 2: Cystic Fibrosis (3 of 5) Lisa, age 7 years, has CF. She lives with both parents and a 4-year-old sister who also has CF. Lisa tells the nurse that she would like to play soccer like my friends. The nurses recommendation should be based on knowledge that physical exercise is: A. important because it encourages effective breathing. B. important because it stimulates underactive sweat glands. C. contraindicated because it causes coughing. D. contraindicated because it causes forced expiration.

A. important because it encourages effective breathing. Rationale: A. Physical exercise is an important adjunct to daily chest physical therapy to maintain pulmonary hygiene. It stimulates mucus secretion and provides a sense of well-being and increased self-esteem in the child. B. Sweat glands are not underactive in CF. C. Coughing is a desired effect of pulmonary hygiene (and exercise) to mobilize secretions from the lungs. D. Forced expiration is also desired to facilitate expectoration of secretions from the lungs.

Steven is a 4-year-old child who is brought to the emergency department by his parents because he is "very sick." Symptoms include a fever of 40° C (104° F), stridor, and sore throat. He is agitated, is drooling, and wants to sit up. Acute epiglottitis is the probable diagnosis. (4 of 4) The nurse should anticipate that the first treatment for epiglottitis will be A. intubation. B. intravenous antibiotics. C. cough suppressant. D. placement in a croup tent

A. intubation. Rationale: A. The primary management goal of epiglottitis is to stabilize the airway; this can be done by intubation or tracheostomy. B. Intravenous antibiotics are usually administered after the child's airway is stabilized. Starting an intravenous line causes pain, which can further compromise the child's airway. C. A cough is not a common symptom of epiglottitis; the child should not be asked to swallow any medication until his airway is stabilized. D. The child should be allowed to remain in the position that provides the most comfort and security, usually a sitting position held by the parents. Humidity is not an intervention in the treatment of acute epiglottitis.

Case 2: Cystic Fibrosis (4 of 5) Lisa, age 7 years, has CF. She lives with both parents and a 4-year-old sister who also has CF. Lisa's father calls the clinic and tells the nurse that Lisa seems very sick. Lisas father describes her symptoms as breathing very fast, fast heart rate, short of breath, pale, and color is bluish. The nurse recognizes these symptoms as A. possible pneumothorax. B. possible bronchospasms. C. terminal stage of the disease. D. normal progression of the disease.

A. possible pneumothorax. Rationale: A. Pneumothorax, air in the pleural space, is often caused by rupture of subpleural blebs through the visceral pleura and usually causes nonspecific symptoms, which include tachypnea, tachycardia, dyspnea, pallor, and cyanosis. B. Symptoms of bronchospasm usually include dyspnea, wheezing, and pallor. Bronchospasm is not a common threat to the child with CF. C. Although pneumothorax occurs in patients with more advanced CF, these symptoms do not indicate a terminal stage of the disease. D. It is estimated that only 5% to 8% of all CF patients will eventually have a pneumothorax; therefore, this is not a necessary progression of the disease

Case #3: Bronchiolitis (2 of 6) Jessica is a 6-week-old infant who was born prematurely and weighed 1.8 kg (4 pounds) at birth. Jessica has progressed steadily in weight and other aspects of her development. Her parents are in their thirties and live in a middle-class neighborhood. In addition, they have a 4-year-old son Sam who attends preschool three mornings a week. Recently, Sam has had an upper respiratory tract infection, and now Jessica has been hospitalized with respiratory symptoms that are suggestive of bronchiolitis. Compare assessment differences between bronchiolitis and acute spasmodic laryngitis

Acute spasmodic laryngitis is a croup syndrome with paroxysmal attacks of laryngeal obstruction that occur chiefly at night; it is usually caused by a viral agent. Bronchiolitis is an acute viral infection that occurs primarily in the winter and spring. The symptoms begin with rhinorrhea and fever, often spreading to a lower respiratory tract infection.

Mr. and Mrs. Foster have come to a genetics clinic because she is 40 years old and pregnant for the first time. They seek advice regarding Down syndrome (DS). Provide answers to the following questions about Down syndrome that the Fosters ask. (2 of 4 questions) Is there a test that can provide a prenatal diagnosis of DS?

Amniocentesis with chromosomal analysis or chorionic villus sampling with chromosomal analysis Rationale: These tests allow for chromosomal analysis of fetal cells, which can detect the presence of trisomy or translocation.

Case 2: Cystic Fibrosis (1 of 5) Lisa, age 7 years, has CF. She lives with both parents and a 4-year-old sister who also has CF. Lisa's mother tearfully tells the nurse that she is pregnant and worried that this child will also have CF. Based on the nurses knowledge of genetics and CF, the nurse understands that A. CF is usually not inherited. B. CF can be diagnosed prenatally. C. there is a 50% chance this child will be affected. D. there is a 100% chance this child will be affected if it is a female.

B. CF can be diagnosed prenatally Rationale: A. CF is inherited as an autosomal recessive trait; the affected child inherits the defective gene from both parents. B. Genetic discoveries have allowed for better screening techniques, and prenatal testing continues to be studied. C. The autosomal recessive defective gene is inherited from both parents with an overall incidence of 1:4 (25% chance a child will be affected). D. The autosomal recessive defective gene is inherited from both parents with an overall incidence of 1:4 (25% chance a child will be affected). The defective gene is not sex linked; therefore, CF can occur in any sex.

Case 1: (3 of 4) Kimberly, age 9 years, is brought to the clinic because of acute respiratory distress. She appears anxious and is clinging to her father. The diagnosis is asthma. Instructions for using the MDI include to A. avoid breathing too deeply. B. breathe medication in slowly. C. hold inhaler at about a 45-degree angle to the mouth. D. breathe in through nose and out through mouth

B. breathe medication in slowly Rationale: A deep breath is desired to allow the medication to reach the narrowed airways. A slow, deep breath inspiration held for 5 to 10 seconds will allow the medication to reach the narrow, deep airways. Rapid inspirations cause the medication to move through the unobstructed bronchioles to patent airways, where they are less needed. The inhaler should be held upright with the mouthpiece in the mouth. The inspiration should be through the mouth to draw the medication into the lungs, and expiration should be through the nose.

Case 2: (3 of 5) Bill, age 8 years, is brought to the clinic because of acute respiratory distress. He appears anxious and is clinging to his father. The diagnosis is asthma. Bill needs instructions about using a metered-dose inhaler (MDI). The nurse should explain that the MDI is used to A. improve circulation. B. distribute prescribed medication directly to airways. C. assess severity of breathing difficulty. D. distribute prescribed medication systemically without the need for injection

B. distribute prescribed medication directly to airways Rationale: A. An MCI is a method of medication administration. The result from use of the MDI depends on the medication administered. B. The MDI is a self-contained, hand-held device that allows for intermittent delivery of a specified amount of medication into the airways. C. The severity of asthma symptoms is assessed by conducting pulmonary function tests, often with a peak expiratory flowmeter. D. Although an MDI distributes medication directly into the airways, this does not eliminate the possible need for medications that are given via injection, such as epinephrine.

Steven is a 4-year-old child who is brought to the emergency department by his parents because he is "very sick." Symptoms include a fever of 40° C (104° F), stridor, and sore throat. He is agitated, is drooling, and wants to sit up. Acute epiglottitis is the probable diagnosis. (3 of 4) 3. The physician orders a lateral neck x-ray film. The most important nursing action related to this is to A. ask parents to remain with Steven during x-ray examination. B. request portable x-ray film be taken in the emergency department. C. transport Steven in a sitting position to radiology. D. transport Steven in a supine position to radiology.

B. request portable x-ray film be taken in the emergency department. Rationale: A, C, D. Most practitioners prefer that the child not be transported for examination because of the dangerous nature of the illness and the potential for respiratory failure. B. A portable x-ray film should be taken in the room so that the child does not have to be moved and can continue to be held by a parent; this minimizes the child's anxiety during the procedure

Case 2: (1 of 5) Bill, age 8 years, is brought to the clinic because of acute respiratory distress. He appears anxious and is clinging to his father. The diagnosis is asthma. What symptoms support the diagnosis of moderate persistent asthma in a child over 5 years of age? A. Symptoms two to four times a week B. Frequent nighttime symptoms C. Daily symptoms D. Symptoms less than two times a week

C. Daily symptoms Rationale: A. Anxiety is a symptom of respiratory distress. Therefore, the treatment goal is to relieve the cause of distress rather than the anxiety. B. Inflammation would be treated with a corticosteroid, not epinephrine, which is an α-adrenergic agonist. C. Epinephrine is an α-adrenergic agonist, which causes mucosal vasoconstriction and subsequently relieves bronchospasm. D. Respiratory acidosis would be treated with sodium bicarbonate, rather than epinephrine, which is an α-adrenergic agonist.

Steven is a 4-year-old child who is brought to the emergency department by his parents because he is "very sick." Symptoms include a fever of 40° C (104° F), stridor, and sore throat. He is agitated, is drooling, and wants to sit up. Acute epiglottitis is the probable diagnosis. (2 of 4) Steven's parents ask what causes epiglottitis. Based on the nurse's knowledge of epiglottitis, the most appropriate response is A. viral. B. bacterial, usually Staphylococcus aureus, C. bacterial, usually Haemophilus influenzae. D. bacterial, usually ß-hemolytic streptococci.

C. bacterial, usually Haemophilus influenzae. Rationale: The causative agent of acute epiglottitis is bacterial, usually H. influenzae.

Case 1: (2 of 4) Kimberly, age 9 years, is brought to the clinic because of acute respiratory distress. She appears anxious and is clinging to her father. The diagnosis is asthma. Kimberly needs instructions about using a metered-dose inhaler (MDI). The nurse should explain that the MDI is used to A. improve oxygenation. B. assess severity of symptoms. C. distribute prescribed medication directly to airways. D. distribute prescribed medication systemically without the need for injection

C. distribute prescribed medication directly to airways. Rationale: A. An MDI is a method of medication administration. The result from use of the MDI depends on the medication administered. B. The severity of asthma symptoms is assessed by conducting pulmonary function tests, often with a peak expiratory flowmeter. C. The MDI is a self-contained, hand-held device that allows for intermittent delivery of a specified amount of medication into the airways. D. Although an MDI distributes medication directly into the airways, this does not eliminate the possible need for medications that are given via injection, such as epinephrine.

Case 1: (1 of 4) Kimberly, age 9 years, is brought to the clinic because of acute respiratory distress. She appears anxious and is clinging to her father. The diagnosis is asthma. Epinephrine is ordered to be given subcutaneously. The purpose of epinephrine is to A. promote relaxation. B. reduce inflammation. C. relieve bronchospasm. D. correct respiratory acidosis.

C. relieve bronchospasm Rationale: Anxiety is a symptom of respiratory distress. Therefore, the treatment goal is to relieve the cause of distress rather than the anxiety. Inflammation would be treated with a corticosteroid, not epinephrine, which is an α-adrenergic agonist. Epinephrine is an α-adrenergic agonist, which causes mucosal vasoconstriction and subsequently relieves bronchospasm. Respiratory acidosis would be treated with sodium bicarbonate, rather than epinephrine, which is an α-adrenergic agonist.

Case 2: Cystic Fibrosis (5 of 5) Lisa, age 7 years, has CF. She lives with both parents and a 4-year-old sister who also has CF. When discussing nutrition with Lisa and her family, the nurse recommends continued supplementation of vitamins A, D, E, and K. This is important because A. pancreatic enzymes are administered with meals. B. children with CF cannot receive a well-balanced diet. C. uptake of fat-soluble vitamins is decreased in CF. D. excretion of water-soluble vitamins is increased in CF

C. uptake of fat-soluble vitamins is decreased in CF. Rationale: A. The administration of pancreatic enzymes with meals and snacks is the principle treatment for pancreatic insufficiency and not related to the need for vitamin supplementation. B. Children with CF require a well-balanced, high-protein, high-calorie diet. C. The uptake of fat-soluble vitamins is decreased because of pancreatic insufficiency. D. Vitamins A, D, E, and K are fat-soluble vitamins.

Case 1: Cystic Fibrosis (5 of 5) David is the 3-year-old son of parents who are 38 and 40 years old. Mr. and Mrs. Bradley have been married for 15 years and underwent fertility therapy to have a child. David has a history of frequent colds increasing in severity during the past year. In addition, David has been losing weight, although he eats well. Mrs. Bradley has noticed during the past year that David has been having large bulky, foul-smelling bowel movements. David is admitted to the pediatric unit for a workup for cystic fibrosis (CF). List seven nursing diagnoses appropriate for David.

Correct Answer: 1. Ineffective Airway Clearance related to secretion of thick, tenacious mucus 2. Ineffective Breathing Pattern related to mechanical airway obstruction 3. Altered Nutrition, Less Than Body Requirements, related to inability to digest nutrients 4. High Risk for Infection related to impaired body defenses 5. Activity Intolerance related to imbalance between oxygen supply and demand 6. Altered Growth and Development related to chronic illness 7. Altered Family Processes related to child with chronic illness Rationale: The nurse has the opportunity to help the patient with CF and his or her family from the time of diagnosis, throughout repeated hospitalizations, and in preparation for home care. Nurses should use a multisystem approach to care that takes into consideration the effects of CF on respiratory and gastrointestinal function, as well as the patients and familys well-being.

Case #3: Bronchiolitis (4 of 6) Jessica is a 6-week-old infant who was born prematurely and weighed 1.8 kg (4 pounds) at birth. Jessica has progressed steadily in weight and other aspects of her development. Her parents are in their thirties and live in a middle-class neighborhood. In addition, they have a 4-year-old son Sam who attends preschool three mornings a week. Recently, Sam has had an upper respiratory tract infection, and now Jessica has been hospitalized with respiratory symptoms that are suggestive of bronchiolitis. Repeated episodes of bronchiolitis not associated with RSV in young children require follow-up testing for which disorders?

Correct Answer: Asthma, cystic fibrosis Rationale: Because of the nature of asthma and cystic fibrosis, the airways are more reactive and/or mucus production is increased, causing a perfect medium for bacterial growth. Children with asthma and cystic fibrosis are more likely to display repeated symptoms of bronchiolitis before the diagnosis of a chronic disorder

Case 1: Cystic Fibrosis (1 of 5) David is the 3-year-old son of parents who are 38 and 40 years old. Mr. and Mrs. Bradley have been married for 15 years and underwent fertility therapy to have a child. David has a history of frequent colds increasing in severity during the past year. In addition, David has been losing weight, although he eats well. Mrs. Bradley has noticed during the past year that David has been having large bulky, foul-smelling bowel movements. David is admitted to the pediatric unit for a workup for cystic fibrosis (CF). What genotype would Mr. and Mrs. Bradley need to be if their son, David, is diagnosed with CF?

Correct Answer: Each would need to carry the recessive gene for CF. Rationale: CF is inherited as an autosomal recessive gene; the child must inherit the defective gene from both parents.

Case #3: Bronchiolitis (3 of 6) Jessica is a 6-week-old infant who was born prematurely and weighed 1.8 kg (4 pounds) at birth. Jessica has progressed steadily in weight and other aspects of her development. Her parents are in their thirties and live in a middle-class neighborhood. In addition, they have a 4-year-old son Sam who attends preschool three mornings a week. Recently, Sam has had an upper respiratory tract infection, and now Jessica has been hospitalized with respiratory symptoms that are suggestive of bronchiolitis. How is the diagnosis of RSV established?

Correct Answer: Enzyme-linked immunosorbent assay (ELISA) of nasal secretions Rationale: Either the rapid immunofluorescent antibody (IFA) or the ELISA technique for RSV detection can be used. These techniques are rapid and have sensitivities and specificities of about 90%.

Case 1: Cystic Fibrosis (2 of 5) David is the 3-year-old son of parents who are 38 and 40 years old. Mr. and Mrs. Bradley have been married for 15 years and underwent fertility therapy to have a child. David has a history of frequent colds increasing in severity during the past year. In addition, David has been losing weight, although he eats well. Mrs. Bradley has noticed during the past year that David has been having large bulky, foul-smelling bowel movements. David is admitted to the pediatric unit for a workup for cystic fibrosis (CF). If Mr. and Mrs. Bradley ask about fertility therapy contributing to their sons illness, the most appropriate response is:

Correct Answer: Fertility therapy has no relationship with CF, which is an inborn error of metabolism affecting the exocrine glands. Rationale: The CF gene has been discovered on the long arm of chromosome 7 and is a genetic mutation not attributed to external factors.

Case #3: Bronchiolitis (5 of 6) Jessica is a 6-week-old infant who was born prematurely and weighed 1.8 kg (4 pounds) at birth. Jessica has progressed steadily in weight and other aspects of her development. Her parents are in their thirties and live in a middle-class neighborhood. In addition, they have a 4-year-old son Sam who attends preschool three mornings a week. Recently, Sam has had an upper respiratory tract infection, and now Jessica has been hospitalized with respiratory symptoms that are suggestive of bronchiolitis. What type of isolation measures are indicated for health personnel?

Correct Answer: Good hand-washing; gown and glove to prevent cross contamination Rationale: RSV is highly virulent, and health care personnel should take precautions to avoid spreading the virus to uninfected hospital personnel, visitors, and patients in the hospital.

Case 1: Cystic Fibrosis (3 of 5) David is the 3-year-old son of parents who are 38 and 40 years old. Mr. and Mrs. Bradley have been married for 15 years and underwent fertility therapy to have a child. David has a history of frequent colds increasing in severity during the past year. In addition, David has been losing weight, although he eats well. Mrs. Bradley has noticed during the past year that David has been having large bulky, foul-smelling bowel movements. David is admitted to the pediatric unit for a workup for cystic fibrosis (CF). What is the significance of sodium and chloride in diagnosing CF?

Correct Answer: Latest evidence-based research indicates that there is a biochemical abnormality related to a decrease in cellular permeability to chloride; this abnormality is present from birth, with sweat chlorides two to five times greater in affected individuals. Rationale: A clinical feature of CF is a striking elevation of sweat electrolytes. Although sodium and chloride are affected, the defect appears to be primarily a result of abnormal chloride movements.

Case #3: Bronchiolitis (1 of 6) Jessica is a 6-week-old infant who was born prematurely and weighed 1.8 kg (4 pounds) at birth. Jessica has progressed steadily in weight and other aspects of her development. Her parents are in their thirties and live in a middle-class neighborhood. In addition, they have a 4-year-old son Sam who attends preschool three mornings a week. Recently, Sam has had an upper respiratory tract infection, and now Jessica has been hospitalized with respiratory symptoms that are suggestive of bronchiolitis. What is the most prevalent etiologic agent causing bronchiolitis in young infants?

Correct Answer: Respiratory syncytial virus (RSV) Rationale: RSV is responsible for at least 50% of children admitted for bronchiolitis.

Case #3: Bronchiolitis (6 of 6) Jessica is a 6-week-old infant who was born prematurely and weighed 1.8 kg (4 pounds) at birth. Jessica has progressed steadily in weight and other aspects of her development. Her parents are in their thirties and live in a middle-class neighborhood. In addition, they have a 4-year-old son Sam who attends preschool three mornings a week. Recently, Sam has had an upper respiratory tract infection, and now Jessica has been hospitalized with respiratory symptoms that are suggestive of bronchiolitis. Discuss the guidelines for use of ribavirin aerosol therapy.

Correct Answer: Use for infants at high risk because of other abnormalities, especially chronic lung disorders and immunodeficiency; infants less than 6 months old; and severely ill infants. Special precautions such as mask and goggles are required for caregivers; no pregnant personnel should be involved. Rationale: Because of the potential toxic effects of the drug to health care workers and the unclear evidence of the drugs benefit, the American Academy of Pediatrics recommends the preceding criteria for use of ribavirin.

Steven is a 4-year-old child who is brought to the emergency department by his parents because he is "very sick." Symptoms include a fever of 40° C (104° F), stridor, and sore throat. He is agitated, is drooling, and wants to sit up. Acute epiglottitis is the probable diagnosis. (1 of 4) 1. Which should the nurse do first? A. Assess for dehydration B. Obtain a throat culture C. Have Steven lie down to ease respirations D. Have a pediatric tracheostomy set available

D. Have a pediatric tracheostomy set available Rationale: A. The child's symptoms indicate a respiratory emergency rather than a sign of dehydration. B. Examination of the throat is contraindicated until immediate intubation can be provided. A foreign object introduced into the oral cavity can stimulate fear in the child and bronchospasm, which could completely occlude the child's airway. C. The child should be positioned to allow maximum lung expansion and comfort, such as sitting upright and/or leaning forward, not supine. D. A pediatric tracheostomy set should be available in the event the child's airway becomes completely occluded and intubation is not possible

Case 2: Cystic Fibrosis (2 of 5) Lisa, age 7 years, has CF. She lives with both parents and a 4-year-old sister who also has CF. The pulmonary problems associated with CF are related to A. bronchospasms. B. underdevelopment of alveoli. C. decreased ability of chest to expand. D. increased viscosity of bronchial mucus.

D. increased viscosity of bronchial mucus. Rationale: A. Bronchospasms are not common in the patient with CF. B. The structural components of the lungs are usually well formed. C. Chronic bronchial obstruction resulting from viscous secretions usually leads to increased airway resistance. This, however, is a symptom rather than the source of the respiratory problems caused by CF. D. The primary factor responsible for the clinical manifestations of CF is mechanical obstruction caused by the increased viscosity of mucous gland secretions.

Case #1: Cerebral Palsy (2 of 3) Tony is a 2-year-old boy who was born 8 weeks prematurely. He has been diagnosed with spastic cerebral palsy (CP).

Poor head control after 3 months of age Stiff or rigid arms or legs Pushing away or arching back Floppy or limp body posture Inability to sit up without support by 8 months Use of only one side of the body, or only the arms to crawl Behavioral Signs Extreme irritability or crying Failure to smile by 3 months Feeding difficulties Persistent gagging or choking when fed After 6 months of age, tongue pushing soft food out of the mouth

Mr. and Mrs. Foster have come to a genetics clinic because she is 40 years old and pregnant for the first time. They seek advice regarding Down syndrome (DS). Provide answers to the following questions about Down syndrome that the Fosters ask. (3 of 4 questions)

Rationale: A variety of congenital anomalies, sensory problems, and other physical disorders occur in children with DS, including: • Congenital heart defect in 30% to 40% • Renal agenesis • Duodenal atresia • Patella dislocation • Hip subluxation • Atlantoaxial instability • Tracheoesophageal fistula • Hirschsprung disease • Strabismus • Nystagmus • Astigmatism • Myopia • Hyperopia • Altered immune function

Case 2: (2 of 5) Bill, age 8 years, is brought to the clinic because of acute respiratory distress. He appears anxious and is clinging to his father. The diagnosis is asthma. Match the diagnostic test with the correct definition. A. Pulmonary function test B. Peak expiratory flow rate C. Peak expiratory flowmeter D. Skin testing ____ An instrument used to measure PEFR ____ Maximum flow of air that can be forcefully expelled in 1 second ____ Objective measure of lung function ____ Testing for specific allergens using intradermal injections

_C__ An instrument used to measure PEFR _B__ Maximum flow of air that can be forcefully expelled in 1 second _A__ Objective measure of lung function _D__ Testing for specific allergens using intradermal injections

The nurse is talking to the parent of a 13-month-old child. The mother states, "My child does not make noises like 'da' or 'na' like my sister's baby, who is only 9 months old." Which of the following statements by the nurse would be most appropriate to make? a. "I am going to request a referral to a hearing specialist." b. "You should not compare your child to your sister's child." c. "I think your child is fine, but we will check again in 3 months." d. "You should ask other parents what noises their children made at this age."

a. "I am going to request a referral to a hearing specialist." Rationale: By 11 months of age a child should be making well-formed syllables such as 'da' or 'na' and should be referred to a specialist if not.

An 18-month-old child is seen in the clinic with AOM. Trimethoprim-sulfamethoxazole (Bactrim) is prescribed. Which of the following statements made by the parent indicates a correct understanding of the instructions? a. "I should administer all of the prescribed medication." b. "I should continue medication until the symptoms subside." c. "I will immediately stop giving medication if I notice a change in hearing." d. "I will stop giving medication if fever is still present in 24 hours."

a. "I should administer all of the prescribed medication." Rationale: Antibiotics should be given for their full course to prevent recurrence of infection with resistant bacteria

Hearing is expressed in decibels (dB), or units of loudness. Which of the following is, in decibels, the softest sound a normal ear can hear? a. 0 b. 10 c. 40 to 50 d. 100

a. 0 Ratioanle: By definition, 0 dB is the softest sound the normal ear can hear.

The Heimlich maneuver is recommended for airway obstruction in children older than: a. 1 year. b. 4 years. c. 8 years. d. 12 years.

a. 1 year. Rationale: The Heimlich maneuver is recommended for airway obstruction in children older than 1 year. Younger than 1 year, back blows and chest thrusts are administered.

Which of the following types of seizures may be difficult to detect? a. Absence b. Generalized c. Simple partial d. Complex partial

a. Absence Rationale: Absence seizures may go unrecognized because little change occurs in the child's behavior during the seizure.

Which statement is most accurate in describing tetanus? a. Acute infectious disease caused by an exotoxin produced by an anaerobic, gram-positive bacillus b. Inflammatory disease that causes extreme, localized muscle spasm c. Acute infection that causes meningeal inflammation, resulting in symptoms of generalized muscle spasm d. Disease affecting the salivary gland with resultant stiffness of the jaw

a. Acute infectious disease caused by an exotoxin produced by an anaerobic, gram-positive bacillus.

When should children with cognitive impairment be referred for stimulation and educational programs? a. As young as possible b. As soon as they have the ability to communicate in some way c. At age 3 years, when schools are required to provide services d. At age 5 or 6 years, when schools are required to provide services

a. As young as possible Rationale: The child's education should begin as soon as possible. Considerable evidence exists that early intervention programs for children with disabilities are valuable for cognitively impaired children.

A child has a chronic, nonproductive cough and diffuse wheezing during the expiratory phase of respiration. This suggests which of the following? a. Asthma b. Pneumonia c. Bronchiolitis d. Foreign body in trachea

a. Asthma Rationale: Asthma may have these chronic symptoms.

An infant's parents ask the nurse about preventing OM. Which of the following should be recommended? a. Avoid tobacco smoke. b. Use nasal decongestant. c. Avoid children with OM. d. Bottle-feed or breastfeed in supine position.

a. Avoid tobacco smoke. Rationale: Eliminating tobacco smoke from the child's environment is essential for preventing OM and other common childhood illnesses.

Latex allergy is suspected in a child with spina bifida. Appropriate nursing interventions include which of the following? a. Avoid using any latex product. b. Use only nonallergenic latex products. c. Administer medication for long-term desensitization. d. Teach family about long-term management of asthma.

a. Avoid using any latex product. Rationale: Care must be taken that individuals who are at high risk for latex allergies do not come in direct or secondary contact with products or equipment containing latex at any time during medical treatment.

Decongestant nose drops are recommended for a 10-month-old infant with an upper respiratory tract infection. Instructions for nose drops should include which of the following? a. Avoid using for more than 3 days. b. Keep drops to use again for nasal congestion. c. Administer drops until nasal congestion subsides. d. Administer drops after feedings and at bedtime.

a. Avoid using for more than 3 days Rationale: Vasoconstrictive nose drops such as phenylephrine (Neo-Synephrine) should not be used for more than 3 days to avoid rebound congestion.

Prevention of hearing impairment in children is a major goal for the nurse. This can be achieved through which of the following? a. Being involved in immunization clinics for children b. Assessing a newborn for hearing loss c. Answering parents' questions about hearing aids d. Participating in hearing screening in the community

a. Being involved in immunization clinics for children Rationale: Childhood immunizations can eliminate the possibility of acquired sensorineural hearing loss from rubella, mumps, or measles encephalitis.

An adolescent boy is brought to the emergency department after a motorcycle accident. His respirations are deep, periodic, and gasping. There are extreme fluctuations in blood pressure. Pupils are dilated and fixed. The nurse should suspect what type of head injury? a. Brainstem b. Skull fracture c. Subdural hemorrhage d. Epidural hemorrhage

a. Brainstem Rationale: Signs of brainstem injury include deep, rapid, periodic or intermittent, and gasping respirations. Wide fluctuations or noticeable slowing of the pulse, widening pulse pressure, or extreme fluctuations in blood pressure are consistent with a brainstem injury.

A school-age child had an upper respiratory tract infection for several days and then began having a persistent dry, hacking cough that was worse at night. The cough has become productive in the past 24 hours. This is most suggestive of which of the following? a. Bronchitis b. Bronchiolitis c. Viral-induced asthma d. Acute spasmodic laryngitis

a. Bronchitis Rationale: Bronchitis is characterized by these symptoms and occurs in children older than 6 years.

What clinical manifestations would suggest hydrocephalus in a neonate? a. Bulging fontanel and dilated scalp veins b. Closed fontanel and high-pitched cry c. Constant low-pitched cry and restlessness d. Depressed fontanel and decreased blood pressure

a. Bulging fontanel and dilated scalp veins Rationale: Bulging fontanels, dilated scalp veins, and separated sutures are clinical manifestations of hydrocephalus in neonates.

Apnea of infancy has been diagnosed in an infant scheduled for discharge with home monitoring. Part of the infant's discharge teaching plan should include? a. Cardiopulmonary resuscitation (CPR) b. Administration of intravenous (IV) fluids c. Foreign airway obstruction removal using the Heimlich maneuver d. Advice that the infant not be left with caretakers other than the parents

a. Cardiopulmonary resuscitation (CPR)

An infant has developed staphylococcal pneumonia. Nursing care of the child with pneumonia includes which of the following? (Select all that apply.) a. Cluster care to conserve energy b. Round-the-clock administration of antitussive agents c. Strict intake and output to avoid congestive heart failure d. Administration of antibiotics

a. Cluster care to conserve energy d. Administration of antibiotics Rationale: Antibiotics are indicated for a bacterial pneumonia. Often the child will have decreased pulmonary reserve, and the clustering of care is essential.

An important nursing intervention when caring for a child who is experiencing a seizure would be which of the following? a. Describe and record the seizure activity observed. b. Restrain the child when seizure occurs to prevent bodily harm. c. Place a tongue blade between the teeth if they become clenched. d. Suction the child during a seizure to prevent aspiration.

a. Describe and record the seizure activity observed Rationale: When a child is having a seizure, the priority nursing care is observation of the child and seizure. The nurse then describes and records the seizure activity.

Which of the following best describes a neuroblastoma? a. Diagnosis is usually made after metastasis occurs. b. Early diagnosis is usually possible because of the obvious clinical manifestations. c. It is the most common brain tumor in young children. d. It is the most common benign tumor in young children.

a. Diagnosis is usually made after metastasis occurs. Rationale: Neuroblastoma is a silent tumor with few symptoms. In more than 70% of cases, diagnosis is made after metastasis occurs, with the first signs caused by involvement in the nonprimary site.

In providing nourishment for a child with CF, which of the following factors should the nurse keep in mind? a. Diet should be high in carbohydrates and protein. b. Diet should be high in easily digested carbohydrates and fats. c. Most fruits and vegetables are not well tolerated. d. Fats and proteins must be greatly curtailed.

a. Diet should be high in carbohydrates and protein. Rationale: Children with CF require a well-balanced, high-protein, high-calorie diet because of impaired intestinal absorption.

Which of the following is the priority nursing intervention when a child is unconscious after a fall? a. Establish adequate airway. b. Perform neurologic assessment. c. Monitor intracranial pressure. d. Determine whether a neck injury is present.

a. Establish adequate airway. Rationale: Respiratory effectiveness is the primary concern in the care of the unconscious child. Establishment of an adequate airway is always the first priority.

A nurse would suspect possible visual impairment in a child who displays which of the following? a. Excessive rubbing of the eyes b. Rapid lateral movement of the eyes c. Delay in speech development d. Lack of interest in casual conversation with peers

a. Excessive rubbing of the eyes Rationale: Excessive rubbing of the eyes is a clinical manifestation of visual impairment.

Which of the following problems is most often associated with myelomeningocele? a. Hydrocephalus b. Craniosynostosis c. Biliary atresia d. Esophageal atresia

a. Hydrocephalus Rationale: Hydrocephalus is a frequently associated anomaly in 80% to 90% of children

Spastic cerebral palsy is characterized by which of the following? a. Hypertonicity and poor control of posture, balance, and coordinated motion b. Athetosis and dystonic movements c. Wide-based gait and poor performance of rapid, repetitive movements d. Tremors and lack of active movement

a. Hypertonicity and poor control of posture, balance, and coordinated motion Rationale: Hypertonicity and poor control of posture, balance, and coordinated motion are part of the classification of spastic cerebral palsy.

Which statement expresses accurately the genetic implications of cystic fibrosis (CF)? a. If it is present in a child, both parents are carriers of this defective gene. b. It is inherited as an autosomal dominant trait. c. It is a genetic defect found primarily in non-Caucasian population groups. d. There is a 50% chance that siblings of an affected child also will be affected.

a. If it is present in a child, both parents are carriers of this defective gene. Rationale: CF is an autosomal recessive gene inherited from both parents.

The nurse is planning care for a school-age child with bacterial meningitis. Which nursing intervention should be included? a. Keep environmental stimuli to a minimum. b. Avoid giving pain medications that could dull the sensorium. c. Measure the head circumference to assess developing complications. d. Have the child move the head side to side at least every 2 hours.

a. Keep environmental stimuli to a minimum.

Autism is a complex developmental disorder. The diagnostic criteria for autism include delayed or abnormal functioning in which of the following areas with onset before age 3 years? (Select all that apply.) a. Language as used in social communication b. Parallel play c. Gross motor development d. Growth below the 5th percentile for height and weight e. Symbolic or imaginative play f. Social interaction

a. Language as used in social communication e. Symbolic or imaginative play f. Social interaction Rationale: These are three of the areas in which autistic children may show delayed or abnormal functioning.

Which of the following drugs would be used to treat a child who has increased ICP resulting from cerebral edema? a. Mannitol b. Epinephrine hydrochloride c. Atropine sulfate d. Sodium bicarbonate

a. Mannitol Rationale: For increased ICP, mannitol, an osmotic diuretic, administered intravenously, is the drug used most frequently for rapid reduction.

What is the earliest recognizable clinical manifestation(s) of CF? a. Meconium ileus b. History of poor intestinal absorption c. Foul-smelling, frothy, greasy stools d. Recurrent pneumonia and lung infections

a. Meconium ileus Rationale: The earliest clinical manifestation of CF is a meconium ileus, which is found in about 10% of children with CF. Clinical manifestations include abdominal distention, vomiting, failure to pass stools, and rapid development of dehydration.

When caring for the child with Reye syndrome, the priority nursing intervention would be which of the following? a. Monitor intake and output. b. Prevent skin breakdown. c. Observe for petechiae. d. Do range-of-motion exercises.

a. Monitor intake and output. Rationale: Accurate and frequent monitoring of intake and output is essential for adjusting fluid volumes to prevent both dehydration and cerebral edema.

A 14-year-old girl is in the intensive care unit after a spinal cord injury 2 days ago. Nursing care for this child includes which of the following? (Select all that apply.) a. Monitoring and maintaining systemic blood pressure b. Administering corticosteroids c. Minimizing environmental stimuli d. Discussing long-term care issues with the family e. Monitoring for respiratory complications

a. Monitoring and maintaining systemic blood pressure b. Administering corticosteroids e. Monitoring for respiratory complications Rationale: Spinal cord injury patients are physiologically labile, and close monitoring is required. They may be unstable for the first few weeks after the injury. Corticosteroids are administered to minimize the inflammation present with the injury

Which of the following terms refers to the ability to see objects clearly at close range but not at a distance? a. Myopia b. Amblyopia c. Cataract d. Glaucoma

a. Myopia Rationale: Myopia, or nearsightedness, refers to the ability to see objects clearly at close range but not a distance.

Which of the following is the most common problem of children born with a myelomeningocele? a. Neurogenic bladder b. Mental retardation c. Respiratory compromise d. Cranioschisis

a. Neurogenic bladder Rationale: Myelomeningocele is one of the most common causes of neuropathic (neurogenic) bladder dysfunction among children.

The nurse is caring for a 5-year-old child who is scheduled for a tonsillectomy in 2 hours. Which of the following actions should the nurse include in the child's postoperative care plan? (Select all that apply.) a. Notify the surgeon if the child swallows frequently. b. Apply a heat collar to the child for pain relief. c. Place the child on the abdomen until fully wake. d. Allow the child to have diluted juice after the procedure. e. Encourage the child to cough frequently.

a. Notify the surgeon if the child swallows frequently. c. Place the child on the abdomen until fully wake. d. Allow the child to have diluted juice after the procedure. Rationale: Frequent swallowing is a sign of bleeding in children after a tonsillectomy. The child should be placed on the abdomen or the side to facilitate drainage. The child can drink diluted juice, cool water, or popsicles after the procedure.

An infant with hydrocephalus is hospitalized for surgical placement of a ventriculoperitoneal shunt. Which of the following interventions should be included in the child's postoperative care? (Select all that apply.) a. Observe closely for signs of infection. b. Pump the shunt reservoir to maintain patency. c. Administer sedation to decrease irritability. d. Maintain Trendelenburg position to decrease pressure on the shunt. e. Maintain an accurate record of intake and output. f. Monitor for abdominal distention.

a. Observe closely for signs of infection. e. Maintain an accurate record of intake and output. f. Monitor for abdominal distention. Rationale: Infection is a major complication of ventriculoperitoneal shunts. Observation for signs of infection is a priority nursing intervention. Intake and output should be measured carefully. Abdominal distention could be a sign of peritonitis or a postoperative ileus.

Which of the following tests is never performed on an awake child? a. Oculovestibular response b. Doll's head maneuver c. Funduscopic examination for papilledema d. Assessment of pyramidal tract lesions

a. Oculovestibular response Rationale: The oculovestibular response (caloric test) involves the instillation of ice water into the ear of a comatose child. The caloric test is painful and is never performed on an awake child or one who has a ruptured tympanic membrane.

Effective CPR on a 5-year-old child should include which of the following? a. One breath to every five chest compressions b. Two breaths to every 15 chest compressions c. Reassessment of child after 50 cycles of compression and ventilation d. Reassessment of child every 10 minutes that CPR continues

a. One breath to every five chest compressions Rationale: One breath to five chest compressions is the standard for infants.

A 3-year-old child is status post shunt revision for hydrocephaly. Part of the discharge teaching plan for the parents is signs of shunt malformation. Which signs are of shunt malformation? (Select all that apply.) a. Personality change b. Bulging anterior fontanel c. Vomiting d. Dizziness e. Fever

a. Personality change c. Vomiting e. Fever

The nurse enters a room and finds a 6-year-old child who is unconscious. After calling for help and before being able to use an automatic external defibrillator, what steps should the nurse take? Place in correct order. a. Place on a hard surface. b. Administer 30 chest compressions with two breaths. c. Feel carotid pulse while maintaining head tilt with the other hand. d. Use the head tilt-chin lift maneuver and check for breathing. e. Place heel of one hand on lower half of sternum with other hand on top. f. Give two rescue breaths.

a. Place on a hard surface. d. Use the head tilt-chin lift maneuver and check for breathing. f. Give two rescue breaths. c. Feel carotid pulse while maintaining head tilt with the other hand. e. Place heel of one hand on lower half of sternum with other hand on top. b. Administer 30 chest compressions with two breaths.

The nurse is doing a neurologic assessment on a child whose level of consciousness has been variable since sustaining a cervical neck injury 12 hours ago. What is the priority assessment for this child? a. Reactivity of pupils b. Doll's head maneuver c. Oculovestibular response d. Funduscopic examination to identify papilledema

a. Reactivity of pupils

A young boy has just been diagnosed with pseudohypertrophic (Duchenne) muscular dystrophy. The management plan should include which of the following? a. Recommend genetic counseling. b. Explain that the disease is easily treated. c. Suggest ways to limit use of muscles. d. Assist family in finding a nursing facility to provide his care.

a. Recommend genetic counseling. Rationale: Pseudohypertrophic (Duchenne) muscular dystrophy is inherited as an X-linked recessive gene. Genetic counseling is recommended for parents, female siblings, maternal aunts, and their female offspring.

Which of the following should be the major consideration when selecting toys for a child who is mentally retarded? a. Safety b. Age appropriateness c. Ability to provide exercise d. Ability to teach useful skills

a. Safety Rationale: Safety is the primary concern in selecting recreational and exercise activities for all children. This is especially true for children who are mentally retarded.

What is a clinical manifestation of increased intracranial pressure (ICP) in infants? a. Shrill, high-pitched cry b. Photophobia c. Pulsating anterior fontanel d. Vomiting and diarrhea

a. Shrill, high-pitched cry

Which of the following facilitates lip reading by the hearing-impaired child? a. Speak at an even rate. b. Exaggerate pronunciation of words. c. Avoid using facial expressions. d. Repeat in exactly the same way if child does not understand.

a. Speak at an even rate. Rationale: The child should be helped to learn and understand how to read lips by speaking at an even rate.

A 10-year-old child, without a history of previous seizures, experiences a tonic-clonic seizure at school. Breathing is not impaired, but some postictal confusion occurs. The most appropriate initial action by the school nurse is which of the following? a. Stay with child and have someone call emergency medical service (EMS). b. Notify parent and regular practitioner. c. Notify parent that child should go home. d. Stay with child, offering calm reassurance.

a. Stay with child and have someone call emergency medical service (EMS). Rationale: The EMS should be called to transport the child because this is the child's first seizure.

The treatment of brain tumors in children consists of which of the following therapies? (Select all that apply.) a. Surgery b. Bone marrow transplantation c. Chemotherapy d. Stem cell transplantation e. Radiation f. Myelography

a. Surgery c. Chemotherapy e. Radiation Rationale: Treatment for brain tumors in children may consist of surgery, chemotherapy, and radiotherapy alone or in combination.

Which of the following tests aid in the diagnosis of CF? a. Sweat chloride test, stool for fat, chest radiograph films b. Stool for fat, gastric contents for hydrochloride, chest radiograph films c. Sweat chloride test, bronchoscopy, duodenal fluid analysis d. Sweat chloride test, stool for trypsin, biopsy of intestinal mucosa

a. Sweat chloride test, stool for fat, chest radiograph films Rationale: A sweat test result of greater than 60 mEq/L is diagnostic of CF, a high level of fecal fat is a gastrointestinal (GI) manifestation of CF, and a chest radiograph showing patchy atelectasis and obstructive emphysema indicates CF.

A 6-year-old child born with a myelomeningocele has a neurogenic bladder. The parents have been performing clean intermittent catheterization. What should the nurse recommend? a. Teach the child to do self-catheterization. b. Teach the child appropriate bladder control. c. Continue having the parents do the catheterization. d. Encourage the family to consider urinary diversion

a. Teach the child to do self-catheterization

Which of the following statements is the most descriptive of bronchial asthma? a. There is heightened airway reactivity. b. There is decreased resistance in the airway. c. The single cause of asthma is an allergic hypersensitivity. d. It is inherited.

a. There is heightened airway reactivity. Rationale: In bronchial asthma, spasm of the smooth muscle of the bronchi and bronchioles causes constriction, producing impaired respiratory function.

Why are cool-mist vaporizers rather than steam vaporizers recommended in home treatment of respiratory tract infections? a. They are safer. b. They are less expensive. c. Respiratory secretions are dried. d. A more comfortable environment is produced.

a. They are safer. Rationale: Cool-mist vaporizers are safer than steam vaporizers, and little evidence exists to show any advantages to steam.

A 5-year-old child is brought the Emergency Department with abrupt onset of sore throat, pain with swallowing, fever, and sitting upright and forward. Acute epiglottitis is suspected. What are the most appropriate nursing interventions? (Select all that apply.) a. Vital signs b. Throat culture c. Medical history d. Assessment of breath sounds e. Emergency airway equipment readily available

a. Vital signs c. Medical history d. Assessment of breath sounds e. Emergency airway equipment readily available

A child with CF receives aerosolized bronchodilator medication. This medication should be administered: a. before chest physiotherapy (CPT). b. after CPT. c. before receiving 100% oxygen. d. after receiving 100% oxygen.

a. before chest physiotherapy (CPT). Rationale: Bronchodilators should be given before CPT to open bronchi and make expectoration easier.

The most common type of hearing loss, which results from interference of transmission of sound to the middle ear, is called: a. conductive. b. sensorineural. c. mixed conductive-sensorineural. d. central auditory imperceptive.

a. conductive Rationale: Conductive or middle-ear hearing loss is the most common type. It results from interference of transmission of sound to the middle ear, most often from recurrent otitis media.

Labored breathing is referred to as: a. dyspnea. b. tachypnea. c. hypopnea. d. orthopnea.

a. dyspnea. Rationale: Dyspnea is labored breathing.

Asthma is classified into four categories: mild intermittent, mild persistent, moderate persistent, and severe persistent. Clinical features used to determine these categories include (Select all that apply) a. lung function. b. associated allergies. c. frequency of symptoms. d. frequency and severity of exacerbations.

a. lung function. c. frequency of symptoms. d. frequency and severity of exacerbations.

A toddler fell out of a second-story window. She had brief loss of consciousness and vomited four times. Since admission, she has been alert and oriented. Her mother asks why a CT scan is required when she "seems fine." The nurse should explain that she: a. may have a brain injury. b. needs this because of her age. c. may start having seizures. d. probably has a skull fracture.

a. may have a brain injury. Rationale: The child's history of the fall, brief loss of consciousness, and vomiting four times necessitates evaluation of a potential brain injury. The severity of a head injury may not be apparent on clinical examination but will be detectable on a CT scan.

A 15 year-old is admitted to the intensive care unit (ICU) with a spinal cord injury. The most appropriate nursing interventions for this adolescent are (select all that apply) a. monitoring neurologic status. b. administering corticosteroids. c. monitoring for respiratory complications. d. discussing long-term care issues with the family. e. monitoring and maintaining hemodynamic status.

a. monitoring neurologic status. b. administering corticosteroids. c. monitoring for respiratory complications. e. monitoring and maintaining hemodynamic status.

The nurse is performing a neurologic assessment of a 2-month-old infant after a car accident. Moro, tonic neck, and withdrawal reflexes are present. The nurse should recognize that these reflexes suggest a. neurologic health b. severe brain damage c. decorticate posturing d. decerebrate posturing

a. neurologic health

The nurse is caring for a toddler who has had surgery for a brain tumor. During an assessment, the nurse notes that the child is becoming irritable and the pupils are unequal and sluggish. The most appropriate nursing action is to a. notify the practitioner immediately. b. assess for level of consciousness (LOC). c. observe closely for signs of increased intracranial pressure (ICP). d. administer pain medication and assess for response.

a. notify the practitioner immediately

The parent of a child with cystic fibrosis calls the clinic nurse to report that the child has developed tachypnea, tachycardia, dyspnea, pallor, and cyanosis. The nurse should tell the parent to bring the child to the clinic because these symptoms are suggestive of a. pneumothorax. b. bronchodilation. c. carbon dioxide retention. d. increased viscosity of sputum.

a. pneumothorax.

An infant with a congenital heart defect is receiving palivizumab (Synagis). Based on the nurse's knowledge of medication, the purpose of this medication is to a. prevent respiratory syncytial virus (RSV) infection. b. make isolation of the infant with RSV unnecessary. c. prevent secondary bacterial infection. d. decrease toxicity of antiviral agents.

a. prevent respiratory syncytial virus (RSV) infection.

A 6-year-old has difficulty hearing faint or distant speech. The child's speech is normal, but the child is having problems with school performance. This hearing loss would most likely be classified as a. slight. b. severe. c. moderate. d. inattentiveness rather than hearing loss.

a. slight.

A 4-year-old boy needs to use a metered-dose inhaler to treat asthma. He cannot coordinate the breathing to use it effectively. The nurse should suggest that he use a a. spacer. b. nebulizer. c. peak expiratory flow meter. d. trial of chest physiotherapy.

a. spacer.

The most appropriate nursing intervention for a child following a tonsillectomy is to a. watch for continuous swallowing. b. encourage gargling to reduce discomfort. c. position the child on the back for sleeping. d. apply warm compresses to the throat.

a. watch for continuous swallowing.

The nurse is preparing a school-age child for computed tomography (CT scan) to assess cerebral function. The nurse should include which of the following statements in preparing the child? a. "Pain medication will be given." b. "The scan will not hurt." c. "You will be able to move once the equipment is in place." d. "Unfortunately no one can remain in the room with you during the test."

b. "The scan will not hurt." Rationale: For CT scans, the child must be immobilized. It is important to emphasize to the child that at no time is the procedure painful.

How much folic acid is recommended for women of childbearing age? a. 1.0 mg b. 0.4 mg c. 1.5 mg d. 2.0 mg

b. 0.4 mg Rationale: It has been estimated that a daily intake of 0.4 mg of folic acid in women of childbearing age will prevent 50% to 70% of cases of neural tube defects.

The nurse should suspect a hearing impairment in an infant who demonstrates which of the following behaviors? a. Absence of the Moro reflex b. Absence of babbling by age 7 months c. Lack of eye contact when being spoken to d. Lack of gesturing to indicate wants after age 15 months

b. Absence of babbling by age 7 months Rationale: The absence of babbling or inflections in voice by age 7 months is an indication of hearing difficulties.

When a child with mild mental retardation reaches the end of adolescence, which of the following characteristics would be expected? a. Achieves a mental age of 5 to 6 years b. Achieves a mental age of 8 to 12 years c. Unable to progress in functional reading or arithmetic d. Acquires practical skills and useful reading and arithmetic to an eighth-grade level

b. Achieves a mental age of 8 to 12 years Rationale: By the end of adolescence, the child with mild mental retardation can acquire practical skills and useful reading and arithmetic to a third- to sixth-grade level. A mental age of 8 to 12 years is obtainable, and the child can be guided toward social conformity.

The school nurse is caring for a child with a penetrating eye injury. Emergency treatment includes which of the following? a. Apply a regular eye patch. b. Apply a Fox shield to affected eye and any type of patch to the other eye. c. Apply ice until the physician is seen. d. Irrigate eye copiously with a sterile saline solution.

b. Apply a Fox shield to affected eye and any type of patch to the other eye Rationale: The nurse's role in a penetrating eye injury is to prevent further injury to the eye. A Fox shield (if available) should be applied to the injured eye, and a regular eye patch to the other eye to prevent bilateral movement.

Which of the following statements best describes a subdural hematoma? a. Bleeding occurs between the dura and the skull. b. Bleeding occurs between the dura and the cerebrum. c. Bleeding is generally arterial, and brain compression occurs rapidly. d. The hematoma commonly occurs in the parietotemporal region.

b. Bleeding occurs between the dura and the cerebrum Rationale: A subdural hematoma is bleeding that occurs between the dura and the cerebrum as a result of a rupture of cortical veins that bridge the subdural space.

Cardiopulmonary resuscitation (CPR) is begun on a toddler. Which of the following pulses is usually palpated because it is the most central and accessible? a. Radial b. Carotid c. Femoral d. Brachial

b. Carotid Rationale: In a toddler, the carotid pulse is palpated.

Which of the following is the most important consideration in managing TB in children? a. Skin testing b. Chemotherapy c. Adequate nutrition d. Adequate hydration

b. Chemotherapy Rationale: Drug therapy for TB includes isoniazid, rifampin, and pyrazinamide daily for 2 months and two or three times a week for the remaining 4 months.

Mark, a 9 year old with Down syndrome, is mainstreamed into a regular third grade for part of the school day. His mother asks the school nurse about programs, such as Cub Scouts, that he might join. The nurse's recommendation should be based on which of the following? a. Programs like Cub Scouts are inappropriate for children who are mentally retarded. b. Children with Down syndrome have the same need for socialization as other children. c. Children with Down syndrome socialize better with children who have similar disabilities. d. Parents of children with Down syndrome encourage programs, such as scouting, because they deny that their children have disabilities.

b. Children with Down syndrome have the same need for socialization as other children. Rationale: Children of all ages need peer relationships. Children with Down syndrome should have peer experiences similar to those of other children, such as group outings, Cub Scouts, and Special Olympics.

Which of the following is an implanted ear prosthesis for children with sensorineural hearing loss? a. Hearing aid b. Cochlear implant c. Auditory implant d. Amplification device

b. Cochlear implant Rationale: Cochlear implants are surgically implanted, and they provide a sensation of hearing for individuals who have severe or profound hearing loss of sensorineural origin.

The nurse is assessing a child with croup. Examining the child's throat by using a tongue depressor might precipitate which of the following? a. Inspiratory stridor b. Complete obstruction c. Sore throat d. Respiratory tract infection

b. Complete obstruction Rationale: If a child has acute epiglottitis, examination of the throat may cause complete obstruction and should be performed only when immediate intubation can take place

β-Adrenergic agonists and methylxanthines are often prescribed for a child with an asthma attack. What is their action? a. Liquefy secretions b. Dilate the bronchioles c. Reduce inflammation of the lungs d. Reduce infection

b. Dilate the bronchioles Rationale: These medications work to dilate the bronchioles in acute exacerbations.

A school-age child has sustained a head injury and multiple fractures after being thrown from a horse. The child's level of consciousness is variable. The parents tell the nurse that they think their child is in pain because of periodic crying and restlessness. The most appropriate nursing action is which of the following? a. Discuss with parents the child's previous experiences with pain. b. Discuss with practitioner what analgesia can be safely administered. c. Explain that analgesia is contraindicated with a head injury. d. Explain that analgesia is unnecessary when child is not fully awake and alert.

b. Discuss with practitioner what analgesia can be safely administered. Rationale: A key nursing role is to provide sedation and analgesia for the child. Consultation with the appropriate practitioner is necessary to avoid conflict between the necessity to monitor the child's neurologic status and the promotion of comfort and relief of anxiety.

A newborn assessment shows separated sagittal suture, oblique palpebral fissures, depressed nasal bridge, protruding tongue, and transverse palmar creases. These findings are most suggestive of which of the following? a. Microcephaly b. Down syndrome c. Cerebral palsy d. Fragile X syndrome

b. Down syndrome Rationale: These are characteristics associated with Down syndrome.

Which of the following types of croup is always considered a medical emergency? a. Laryngitis b. Epiglottitis c. Spasmodic croup d. Laryngotracheobronchitis (LTB)

b. Epiglottitis Rationale: Epiglottitis is always a medical emergency needing antibiotics and airway support for treatment.

What is associated with infant botulism? a. Contaminated soil b. Honey and corn syrup c. Commercial infant cereals d. Improperly sterilized bottles

b. Honey and corn syrup

The nurse is caring for a child with carbon monoxide poisoning associated with smoke inhalation. Which of the following is essential in this child's care? a. Monitor pulse oximetry. b. Monitor arterial blood gases. c. Administer oxygen if respiratory distress develops. d. Administer oxygen if child's lips become bright, cherry red.

b. Monitor arterial blood gases. Rationale: Arterial blood gases are the best way to monitor carbon monoxide poisoning.

The nurse is caring for a child with acute respiratory distress syndrome (ARDS) associated with sepsis. Nursing actions should include which of the following? a. Force fluids. b. Monitor pulse oximetry. c. Institute seizure precautions. d. Encourage high-protein diet.

b. Monitor pulse oximet Rationale: Monitoring cardiopulmonary status is an important evaluation tool in the care of the child with ARDS.

The vector reservoir for agents causing viral encephalitis in the United States is which of the following? a. Tarantula spiders b. Mosquitoes and ticks c. Carnivorous wild animals d. Domestic and wild animals

b. Mosquitoes and ticks Rationale: Viral encephalitis, not attributable to a childhood viral disease, is usually transmitted by mosquitoes and ticks. The vector reservoir for most agents pathogenic for humans and detected in the United States are mosquitoes and ticks; therefore most cases of encephalitis appear during the hot summer months.

An 8-year-old child is hospitalized with infectious polyneuritis (Guillain-Barré syndrome). When explaining this disease process to the parents, what should the nurse consider? a. Paralysis is progressive, with little hope for recovery. b. Muscle function will gradually return, and recovery is possible in most children. c. Guillain-Barré syndrome results from an apparently toxic reaction to certain medications. d. Guillain-Barré syndrome is inherited as an autosomal recessive, sex-linked gene.

b. Muscle function will gradually return, and recovery is possible in most children.

The nurse is closely monitoring a child who is unconscious after a fall and notices that the child suddenly has a fixed and dilated pupil. The nurse should interpret this as which of the following? a. Eye trauma b. Neurosurgical emergency c. Severe brainstem damage d. Indication of brain death

b. Neurosurgical emergency Rationale: The sudden appearance of a fixed and dilated pupil(s) is a neurosurgical emergency. The nurse should immediately report this finding.

Appropriate interventions to facilitate socialization of the cognitively impaired child include which of the following? a. Provide age-appropriate toys and play activities. b. Provide peer experiences, such as scouting, when older. c. Avoid exposure to strangers who may not understand cognitive development. d. Emphasize mastery of physical skills because they are delayed more often than verbal skills.

b. Provide peer experiences, such as scouting, when older. Rationale: The acquisition of social skills is a complex task. Children of all ages need peer relationships. Parents should enroll the child in preschool. When older, they should have peer experiences similar to those of other children such as group outings, Boy and Girl Scouts, and Special Olympics.

Which of the following best describes why children have fewer respiratory tract infections as they grow older? a. The amount of lymphoid tissue decreases. b. Repeated exposure to organisms causes increased immunity. c. Viral organisms are less prevalent in the population. d. Secondary infections rarely occur after viral illnesses.

b. Repeated exposure to organisms causes increased immunity. Rationale: Children have increased immunity after exposure to a virus.

Distortion of sound and problems in discrimination are characteristic of what type of hearing loss? a. Conductive b. Sensorineural c. Mixed conductive-sensorineural d. Central auditory imperceptive

b. Sensorineural Rationale: Sensorineural hearing loss, also known as perceptive or nerve deafness, involves damage to the inner ear structures or the auditory nerve. It results in distortion of sounds and problems in discrimination.

Autism is a complex developmental disorder. Diagnostic criteria for autism include delayed or abnormal functioning in which area(s) before 3 years of age? (Select all that apply.) a. Parallel play b. Social interaction c. Gross motor development d. Inability to maintain eye contact e. Language as used in social communication

b. Social interaction d. Inability to maintain eye contact e. Language as used in social communication

Early detection of a hearing impairment is critical because of its effect on a variety of areas of a child's life. Which one is of primary importance? a. Reading development b. Speech development c. Relationships with peers d. Performance at school

b. Speech development

A 3-year-old has cerebral palsy (CP) and is hospitalized for orthopedic surgery. The child's mother states the child has difficulty swallowing and cannot hold a utensil to self-feed. The child is slightly underweight for height. What is the most appropriate nursing action related to feeding? a. Bottle-feed or tube-feed the child with a specialized formula until sufficient weight is gained. b. Stabilize the child's jaw with one hand (either from a front or side position) to facilitate swallowing. c. Place the child in a well-supported, semireclining position to make use of gravity flow. d. Place the child in a sitting position with the neck hyperextended to make use of gravity flow.

b. Stabilize the child's jaw with one hand (either from a front or side position) to facilitate swallowing.

Which of the following terms is used when a patient remains in a deep sleep, responsive only to vigorous and repeated stimulation? a. Coma b. Stupor c. Obtundation d. Persistent vegetative state

b. Stupor Rationale: Stupor exists when the child remains in a deep sleep, responsive only to vigorous and repeated stimulation.

The nurse encourages the mother of a toddler with acute LTB to stay at the bedside as much as possible. The nurse's rationale for this action is primarily which of the following? a. Mothers of hospitalized toddlers often experience guilt. b. The mother's presence will reduce anxiety and ease child's respiratory efforts. c. Separation from mother is a major developmental threat at this age. d. The mother can provide constant observations of the child's respiratory efforts.

b. The mother's presence will reduce anxiety and ease child's respiratory efforts. Rationale: The family's presence will decrease the child's distress.

Five-year-old José is being prepared for surgery to remove a brain tumor. Nursing actions should be based on which of the following? a. Removal of tumor will stop the various symptoms. b. Usually the postoperative dressing covers the entire scalp. c. He is not old enough to be concerned about his head being shaved. d. He is not old enough to understand the significance of the brain.

b. Usually the postoperative dressing covers the entire scalp Rationale: José should be told what he will look and feel like after surgery. This includes the size of the dressing. The nurse can demonstrate on a doll the expected size and shape of the dressing.

When taking the history of a child hospitalized with Reye syndrome, the nurse should not be surprised that a week ago the child had recovered from which of the following? a. Measles b. Varicella c. Meningitis d. Hepatitis

b. Varicella Rationale: Most cases of Reye syndrome follow a common viral illness such as varicella or influenza.

Asthma in infants is usually triggered by: a. medications. b. a viral infection. c. exposure to cold air. d. allergy to dust or dust mites.

b. a viral infection. Rationale: Viral illnesses cause inflammation that causes increased airway reactivity in asthma.

The most appropriate time to perform bronchial postural drainage is a. immediately before all aerosol therapy. b. before meals and at bedtime. c. immediately on arising and at bedtime. d. thirty minutes after meals and at bedtime

b. before meals and at bedtime

The postoperative care of a preschool child who has had a brain tumor removed should include a. recording of colorless drainage as normal on the nurse's notes. b. close supervision of the child while he or she is regaining consciousness. c. positioning the child on the right side in the Trendelenburg position. d. no administration of analgesics.

b. close supervision of the child while he or she is regaining consciousness

The Glasgow Coma Scale consists of an assessment of: a. pupil reactivity and motor response. b. eye opening and verbal and motor responses. c. level of consciousness and verbal response. d. ICP and level of consciousness.

b. eye opening and verbal and motor responses. Rationale: The Glasgow Coma Scale assesses eye opening and verbal and motor responses.

A child with CF is receiving recombinant human deoxyribonuclease (rhDNase). This drug: a. may cause mucus to thicken. b. may cause voice alterations. c. is given subcutaneously. d. is not indicated for children younger than 12 years.

b. may cause voice alterations. Rationale: One of the only adverse effects of DNase is voice alterations and laryngitis.

An immediate intervention to teach parents for when an infant chokes on a piece of food would be to a. have infant lie quietly while a call is placed for emergency help. b. position infant in a head-down, face-down position and administer five quick back slaps. c. administer mouth-to-mouth resuscitation. d. give some water by a cup to relieve the obstruction.

b. position infant in a head-down, face-down position and administer five quick back slaps

The primary goal in caring for the child with cognitive impairment is to a. encourage play. b. promote optimum development. c. help families develop a care plan and have them stay with it. d. develop vocational skills

b. promote optimum development.

The nurse is caring for a neonate born with a myelomeningocele. Surgery to repair the defect is scheduled the next day. The most appropriate way to position and feed this neonate is to place him: a. prone and tube feed. b. prone, turn head to side, and nipple feed. c. supine in infant carrier and nipple feed. d. supine, with defect supported with rolled blankets, and nipple-feed.

b. prone, turn head to side, and nipple feed. Rationale: In the prone position, feeding is a problem. The infant's head is turned to one side for feeding.

A child with spina bifida has developed a latex allergy from numerous bladder catheterizations and surgeries. A priority nursing intervention is to a. recommend allergy testing. b. provide a latex-free environment. c. use only powder-free latex gloves. d. limit the use of latex products as much as possible.

b. provide a latex-free environment.

The mother of a 20-month-old tells the nurse that the child has a barking cough at night. The child's temperature is 37ºC (98.6ºF). The mother states the child is not having difficulty breathing. The nurse suspects croup and should recommend a. controlling the fever with acetaminophen (Tylenol) and call the primary care provider if the cough gets worse tonight. b. trying a cool-mist vaporizer at night and watching for signs of difficulty breathing. c. trying over-the-counter cough medicine and coming to the clinic tomorrow if there is no improvement. d. bringing the child to the hospital to be admitted and to be observed for impending epiglottitis.

b. trying a cool-mist vaporizer at night and watching for signs of difficulty breathing.

A 5-year-old girl sustained a concussion when she fell out of a tree. In preparation for discharge, the nurse is discussing home care with her mother. Which of the following statements made by the mother indicates a correct understanding of the teaching? a. "I should expect my child to have a few episodes of vomiting." b. "If I notice sleep disturbances, I should contact the physician immediately." c. "I should expect my child to have some behavioral changes after the accident." d. "If I notice diplopia, I will have my child rest for 1 hour."

c. "I should expect my child to have some behavioral changes after the accident." Rationale: The parents are advised of probable posttraumatic symptoms that may be expected. These include behavioral changes and sleep disturbances.

A 3-year-old child is hospitalized after a near-drowning accident. The child's mother complains to the nurse, "This seems unnecessary when he is perfectly fine." The nurse's best reply would be which of the following? a. "He still needs a little extra oxygen." b. "I'm sure he is fine, but the doctor wants to make sure." c. "The reason for this is that complications could still occur." d. "It is important to observe for possible central nervous system problems."

c. "The reason for this is that complications could still occur." Rationale: All children who have a near-drowning experience should be admitted to the hospital for observation. Although many children do not appear to have suffered adverse effects from the event, complications such as respiratory compromise and cerebral edema may occur 24 hours after the incident.

A child is brought to the emergency department after experiencing a seizure at school. There is no previous history of seizures. The father tells the nurse that he cannot believe the child has epilepsy. The nurse's best response is which of the following? a. "Epilepsy is easily treated." b. "Very few children have actual epilepsy." c. "The seizure may or may not mean that your child has epilepsy." d. "Your child has had only one convulsion; it probably won't happen again."

c. "The seizure may or may not mean that your child has epilepsy." Rationale: Seizures are the indispensable characteristic of epilepsy; however, not every seizure is epileptic. Epilepsy is a chronic seizure disorder with recurrent and unprovoked seizures.

A child has been seizure free for 2 years. A father asks the nurse how much longer the child will need to take the anti-seizure medications. The nurse includes which of the following in the response? a. Medications can be discontinued at this time. b. The child will need to take the drugs for 5 years after the last seizure. c. A step-wise approach will be used to reduce the dosage gradually. d. Seizure disorders are a life-long problem. Medications cannot be discontinued.

c. A step-wise approach will be used to reduce the dosage gradually. Rationale: A predesigned protocol is used to wean a child gradually off antiseizure medications, usually when the child is seizure free for 2 years and has a normal electroencephalogram (EEG).

Because the absorption of fat-soluble vitamins is decreased in cystic fibrosis, which vitamin supplementation is necessary? a. C, D b. A, E, K c. A, D, E, K d. C, folic acid

c. A, D, E, K

The nurse is admitting a young child to the hospital because bacterial meningitis is suspected. What is the major priority of nursing care? a. Initiate isolation precautions as soon as the diagnosis is confirmed. b. Initiate isolation precautions as soon as the causative agent is identified. c. Administer antibiotic therapy as soon as it is ordered. d. Administer sedatives and analgesics on a preventive schedule to manage pain.

c. Administer antibiotic therapy as soon as it is ordered.

It is generally recommended that a child with acute streptococcal pharyngitis can return to school: a. when sore throat is better. b. if no complications develop. c. after taking antibiotics for 24 hours. d. after taking antibiotics for 3 days.

c. After taking antibiotics for 24 hours. Rationale: After children have taken antibiotics for 24 hours, they are no longer contagious to other children.

A child is diagnosed with influenza, probably type A disease. Management includes which of the following? a. Clear liquid diet for hydration b. Aspirin to control fever c. Amantadine hydrochloride to reduce symptoms d. Antibiotics to prevent bacterial infection

c. Amantadine hydrochloride to reduce symptoms Rationale: Amantadine may reduce symptoms related to influenza A if administered within 24 to 48 hours of onset. It is ineffective against type B or C.

What is defined as reduced visual acuity in one eye despite appropriate optical correction? a. Myopia b. Hyperopia c. Amblyopia d. Astigmatism

c. Amblyopia

A young child's parents call the nurse after their child was bitten by a raccoon in the woods. The nurse's recommendation should be based on which of the following? a. Child should be hospitalized for close observation. b. No treatment is necessary if thorough wound cleaning is done. c. Antirabies prophylaxis must be initiated. d. Antirabies prophylaxis must be initiated if clinical manifestations appear.

c. Antirabies prophylaxis must be initiated. Rationale: Current therapy for a rabid animal bite consists of a thorough cleansing of the wound and passive immunization with human rabies immune globulin (HRIG) as soon as possible

The child with Down syndrome should be evaluated for which of the following before participating in some sports? a. Hyperflexibility b. Cutis marmorata c. Atlantoaxial instability d. Speckling of iris (Brushfield spots)

c. Atlantoaxial instability Rationale: Children with Down syndrome are at risk for atlantoaxial instability. Before participating in sports that put stress on the head and neck, a radiologic examination should be done.

Which of the following neurologic diagnostic tests gives a visualized horizontal and vertical cross section of the brain at any axis? a. Nuclear brain scan b. Echoencephalography c. CT scan d. Magnetic resonance imaging (MRI)

c. CT scan Rationale: A CT scan provides a visualization of the horizontal and vertical cross sections of the brain at any axis.

Which of the following is the most common clinical manifestation of retinoblastoma? a. Glaucoma b. Amblyopia c. Cat's eye reflex d. Sunken eye socket

c. Cat's eye reflex Rationale: When the eye is examined, the light will reflect off of the tumor, giving the eye a whitish appearance. This is called a cat's eye reflex.

Which of the following terms refers to opacity of the crystalline lens that prevents light rays from entering the eye and refracting on the retina? a. Myopia b. Amblyopia c. Cataract d. Glaucoma

c. Cataract Rationale: This is the definition of a cataract.

When assessing the eyes of a neonate, the nurse observes opacity of the lens. This represents which of the following? a. Blindness b. Glaucoma c. Cataracts d. Retinoblastoma

c. Cataracts Rationale: A cataract is opacity of the lens of the eye.

When caring for a newborn with Down syndrome, the nurse should be aware that the most common congenital anomaly associated with Down syndrome is which of the following? a. Hypospadias b. Pyloric stenosis c. Congenital heart disease d. Congenital hip dysplasia

c. Congenital heart disease Rationale: Congenital heart malformations, primarily septal defects, are the most common congenital anomaly in Down syndrome.

Which statement best describes pseudohypertrophic (Duchenne) muscular dystrophy (DMD)? a. DMD is inherited as an autosomal dominant disorder. b. DMD is characterized by weakness of the proximal muscles of both the pelvic and shoulder girdles. c. DMD is characterized by muscle weakness, usually beginning at about age 3 years. d. The onset of DMD occurs in later childhood and adolescence.

c. DMD is characterized by muscle weakness, usually beginning at about age 3 years.

Which of the following types of fractures describes traumatic separation of cranial sutures? a. Basilar b. Compound c. Diastatic d. Depressed

c. Diastatic Rationale: Diastatic skull fractures are traumatic separations of the cranial sutures.

Which of the following is a clinical manifestation of increased intracranial pressure (ICP) in infants? (Select all that apply.) a. Low-pitched cry b. Sunken fontanel c. Diplopia and blurred vision d. Irritability e. Distended scalp veins f. Increased blood pressure

c. Diplopia and blurred vision d. Irritability e. Distended scalp veins Rationale: Diplopia and blurred vision, irritability, and distended scalp veins are signs of increased ICP in infants.

The nurse is caring for a 2-year-old child who is unconscious but stable after a car accident. The child's parents are staying at the bedside most of the time. What is an appropriate nursing intervention? a. Suggest that the parents go home until the child is alert enough to know they are present. b. Use ointment on the lips but do not attempt to cleanse the teeth until swallowing returns. c. Encourage the parents to hold, talk to, and sing to the child as they usually would. d. Position the child with proper body alignment and the head of the bed lowered 15 degrees.

c. Encourage the parents to hold, talk to, and sing to the child as they usually would.

Which of the following drugs is considered the most useful in treating cardiac arrest? a. Bretylium b. Lidocaine hydrochloride c. Epinephrine hydrochloride d. Naloxone (Narcan)

c. Epinephrine hydrochloride Rationale: Epinephrine works on alpha and beta receptors in the heart and is the most useful drug in cardiac arrest.

A recommendation to prevent neural tube defects is the supplementation of which of the following? a. Vitamin A throughout pregnancy b. Multivitamin preparations as soon as pregnancy is suspected c. Folic acid for all women of childbearing age d. Folic acid during the first and second trimesters of pregnancy

c. Folic acid for all women of childbearing age Rationale: The widespread use of folic acid among women of childbearing age has decreased the incidence of spina bifida significantly.

Which of the following types of seizures involves both hemispheres of the brain? a. Focal b. Partial c. Generalized d. Acquired

c. Generalized Rationale: Clinical observations of generalized seizures indicate that the initial involvement is from both hemispheres.

The most common clinical manifestation(s) of brain tumors in children is which of the following? a. Irritability b. Seizures c. Headaches and vomiting d. Fever and poor fine motor control

c. Headaches and vomiting Rationale: Headaches, especially on awakening, and vomiting that is not related to feeding are the most common clinical manifestation(s) of brain tumors in children.

The nurse is discussing sexuality with the parents of an adolescent girl with moderate cognitive impairment. Which of the following should the nurse consider when dealing with this issue? a. Sterilization is recommended for any adolescent with cognitive impairment. b. Sexual drive and interest are limited in individuals with cognitive impairment. c. Individuals with cognitive impairment need a well-defined, concrete code of sexual conduct. d. Sexual intercourse rarely occurs unless the individual with cognitive impairment is sexually abused.

c. Individuals with cognitive impairment need a well-defined, concrete code of sexual conduct. Rationale: Adolescents with moderate cognitive impairment may be easily persuaded and lack judgment. A well-defined, concrete code of conduct with specific instructions for handling certain situations should be laid out for the adolescent.

A father calls the emergency department nurse saying that his daughter's eyes burn after getting some dishwasher detergent in them. The nurse recommends that the child be seen in the emergency department or by an ophthalmologist. The nurse also should recommend which of the following before the child is transported? a. Keep eyes closed. b. Apply cold compresses. c. Irrigate eyes copiously with tap water for 20 minutes. d. Prepare a normal saline solution (salt and water) and irrigate eyes for 20 minutes.

c. Irrigate eyes copiously with tap water for 20 minutes Rationale: The first action is to flush the eyes with clean tap water. This will rinse the detergent from the eyes.

Which of the following is descriptive of a concussion? a. Petechial hemorrhages cause amnesia. b. Visible bruising and tearing of cerebral tissue occur. c. It is a transient and reversible neuronal dysfunction. d. A slight lesion develops remote from the site of trauma.

c. It is a transient and reversible neuronal dysfunction. Rationale: A concussion is a transient, reversible neuronal dysfunction with instantaneous loss of awareness and responsiveness resulting from trauma to the head.

Which of the following is the initial clinical manifestation of generalized seizures? a. Being confused b. Feeling frightened c. Losing consciousness d. Seeing flashing lights

c. Losing consciousness Rationale: Loss of consciousness is a frequent occurrence in generalized seizures and is the initial clinical manifestation.

Which of the following clinical manifestations in an infant would be suggestive of spinal muscular atrophy (Werdnig-Hoffmann disease)? a. Hyperactive deep tendon reflexes b. Hypertonicity c. Lying in the frog position d. Motor deficits on one side of body

c. Lying in the frog position Rationale: The infant lies in the frog position with the legs externally rotated, abducted, and flexed at knees.

A young child who has an intelligence quotient (IQ) of 45 would be described as which of the following? a. Within the lower limits of the range of normal intelligence b. Mildly retarded but educable c. Moderately retarded but trainable d. Severely retarded and completely dependent on others for care

c. Moderately retarded but trainable Rationale: Moderately retarded IQs range between 35 and 55.

An appropriate nursing intervention when caring for an unconscious child would be which of the following? a. Change the child's position infrequently to minimize the chance of increased ICP. b. Avoid using narcotics or sedatives to provide comfort and pain relief. c. Monitor fluid intake and output carefully to avoid fluid overload and cerebral edema. d. Give tepid sponge baths to reduce fever, since antipyretics are contraindicated.

c. Monitor fluid intake and output carefully to avoid fluid overload and cerebral edema. Rationale: Often comatose patients cannot cope with the quantity of fluids that they normally tolerate. Overhydration must be avoided to prevent fatal cerebral edema.

Which of the following terms is used to describe a child's level of consciousness when the child is arousable with stimulation? a. Stupor b. Confusion c. Obtundation d. Disorientation

c. Obtundation Rationale: Obtundation describes a level of consciousness in which the child is arousable with stimulation.

Fragile X syndrome is which of the following? a. Chromosomal defect affecting only females b. Chromosomal defect that follows the pattern of X-linked recessive disorders c. Second most common genetic cause of mental retardation d. Most common cause of noninherited mental retardation

c. Second most common genetic cause of mental retardation Rationale: Fragile X syndrome is the second most common cause of mental retardation after Down syndrome.

It is now recommended that children with asthma who are taking long-term inhaled steroids should be assessed frequently because which of the following may develop? a. Cough b. Osteoporosis c. Slowed growth d. Cushing syndrome

c. Slowed growth Rationale: The growth of children on long-term inhaled steroids should be assessed frequently to assess for systemic effects of these drugs.

What most accurately describes bowel function in children born with a myelomeningocele? a. Incontinence cannot be prevented. b. Enemas and laxatives are contraindicated. c. Some degree of fecal continence can usually be achieved. d. A colostomy is usually required by the time the child reaches adolescence.

c. Some degree of fecal continence can usually be achieved.

A 10-year-old boy on a bicycle has been hit by a car in front of the school. The school nurse immediately assesses airway, breathing, and circulation. The next nursing action should be which of the following? a. Place on side. b. Take blood pressure. c. Stabilize neck and spine. d. Check scalp and back for bleeding.

c. Stabilize neck and spine. Rationale: After determining that the child is breathing and has adequate circulation, the next action is to stabilize the neck and spine to prevent any additional trauma.

The nurse is talking with a 10-year-old boy who wears bilateral hearing aids. The left hearing aid is making an annoying whistling sound that the child cannot hear. Which of the following is the most appropriate nursing action? a. Ignore the sound. b. Ask him to reverse the hearing aids in his ears. c. Suggest he reinsert the hearing aid. d. Suggest he raise the volume of the hearing aid.

c. Suggest he reinsert the hearing aid. Rationale: The whistling sound is acoustic feedback. The nurse should have the child remove the hearing aid and reinsert it, making certain no hair is caught between the ear mold and the ear canal.

A parent whose two school-age children have asthma asks the nurse in what sports, if any, they can participate. The nurse should recommend which of the following? a. Soccer b. Running c. Swimming d. Basketball

c. Swimming Rationale: Swimming is well tolerated in children with asthma because they are breathing air fully saturated with moisture and because of the type of breathing required in swimming.

Which of the following is a progressive infantile spinal muscular atrophy and the most common paralytic form of the floppy infant syndrome? a. Kugelberg-Welander disease b. Charcot-Marie-Tooth disease c. Werdnig-Hoffmann disease d. Duchenne muscular dystrophy

c. Werdnig-Hoffmann disease Rationale: Werdnig-Hoffmann disease (spinal muscular atrophy type 1) is the most common paralytic form of floppy infant syndrome (congenital hypotonia). It is characterized by progressive weakness and wasting of skeletal muscle caused by degeneration of anterior horn cells.

The temperature of an unconscious adolescent is 105º F (40.5º C). The priority nursing intervention is to a. continue to monitor temperature. b. initiate a pain assessment. c. apply a hypothermia blanket. d. administer aspirin stat.

c. apply a hypothermia blanket.

The parents of a cognitively impaired child ask the nurse for guidance with discipline. The most appropriate recommendation by the nurse is that a. discipline is ineffective with cognitively impaired children. b. discipline is not necessary for cognitively impaired children. c. behavior modification is an excellent form of discipline. d. physical punishment is the most appropriate form of discipline.

c. behavior modification is an excellent form of disciplin

The nurse is caring for a comatose child with multiple injuries. The nurse should recognize that pain a. cannot occur if the child is comatose. b. may occur if the child regains consciousness. c. requires astute nursing assessment and management. d. is best assessed by family members who are familiar with the child.

c. requires astute nursing assessment and management.

The nurse is discussing long-term care with the parents of a child who has a ventriculoperitoneal shunt to correct hydrocephalus. An important part of the discussion with the parents is that a. parental protection is essential until the child reaches adulthood. b. mental retardation is to be expected with hydrocephalus. c. shunt malfunction or infection requires immediate treatment. d. most usual childhood activities must be restricted.

c. shunt malfunction or infection requires immediate treatment

A young child is having a seizure that has lasted 35 minutes. There is a loss of consciousness. Based on the nurse's knowledge of seizures, the nurse recognizes this as a. absence seizure. b. generalized seizure. c. status epilepticus. d. simple partial seizure.

c. status epilepticus.

The nurse is talking to a parent with a child who has a latex allergy. Which of the following statements by the parent would indicate a correct understanding of the teaching? a. "My child will have an allergic reaction if he comes in contact with yeast products." b. "My child may have an upset stomach if he eats a food made with wheat or barley." c. "My child will probably develop an allergy to peanuts." d. "My child should not eat bananas or kiwis."

d. "My child should not eat bananas or kiwis." Rationale: There are cross-reactions between latex allergies and a number of foods such as bananas, avocados, kiwi, and chestnuts.

A child is admitted to the pediatric intensive care unit for a submersion injury. The child's parents express guilt over the submersion injury to the nurse. The most appropriate response by the nurse is a. "You will need to watch your child more closely in the future." b. "Why did you let your child almost drown?" c. "Your child will be fine, so don't worry." d. "Tell me more about your feelings."

d. "Tell me more about your feelings."

Chronic otitis media with effusion (OME) is differentiated from acute otitis media (AOM) because it is usually characterized by which of the following? a. Fever as high as 40° C (104° F) b. Severe pain in the ear c. Nausea and vomiting d. A feeling of fullness in the ear

d. A feeling of fullness in the ear Rationale: OME is characterized by feeling of fullness in ear or other nonspecific complaints.

Therapeutic management of a child with tetanus includes the administration of which of the following? a. Nonsteroidal antiinflammatory drugs (NSAIDs) to reduce inflammation b. Muscle stimulants to counteract muscle weakness c. Bronchodilators to prevent respiratory complications d. Antibiotics to control bacterial proliferation at site of injury

d. Antibiotics to control bacterial proliferation at site of injury Rationale: Antibiotics are administered to control the proliferation of the vegetative forms of the organism at the site of infection.

A child with asthma is having pulmonary function tests. What explains the purpose of the peak expiratory flow rate (PEFR)? a. Confirms the diagnosis of asthma b. Determines the cause of asthma c. Identifies the "triggers" of asthma d. Assesses the severity of asthma

d. Assesses the severity of asthma

What nursing intervention is used to prevent increased intracranial pressure (ICP) in an unconscious child? a. Suction the child frequently. b. Provide environmental stimulation. c. Turn the head side to side every hour. d. Avoid activities that cause pain or crying.

d. Avoid activities that cause pain or crying.

Which of the following may be beneficial in reducing the risk of Reye syndrome? a. Immunization against the disease b. Medical attention for all head injuries c. Prompt treatment of bacterial meningitis d. Avoidance of aspirin and ibuprofen for children with varicella or those suspected of having influenza

d. Avoidance of aspirin and ibuprofen for children with varicella or those suspected of having influenza Rationale: Although the etiology of Reye syndrome is obscure, most cases follow a common viral illness, either varicella or influenza. A potential association exists between aspirin therapy and the development of Reye syndrome, so use of aspirin is avoided.

The most common cause of hearing impairment in children is which of the following? a. Auditory nerve damage b. Congenital ear defects c. Congenital rubella d. Chronic otitis media

d. Chronic otitis media Rationale: Chronic otitis media is the most common cause of hearing impairment in children. It is essential that appropriate measures be instituted to treat existing infections and prevent recurrences.

The nurse should recommend medical attention if a child with a slight head injury experiences which of the following? a. Sleepiness b. Vomiting, even once c. Headache, even if slight d. Confusion or abnormal behavior

d. Confusion or abnormal behavior Rationale: Medical attention should be sought if the child exhibits confusion or abnormal behavior, loses consciousness, has amnesia, has fluid leaking from the nose or ears, complains of blurred vision, or has an unsteady gait.

An important nursing consideration when caring for a 10-month-old infant with respiratory syncytial virus (RSV)/bronchiolitis would be which of the following? a. Place in a mist tent. b. Administer antibiotics. c. Administer cough syrup. d. Encourage to drink 8 ounces of formula every 4 hours.

d. Encourage to drink 8 ounces of formula every 4 hours. Rationale:Hydration is very important in children with RSV bronchiolitis to loosen secretions and prevent shock.

The nurse is caring for a 10-month-old infant with respiratory syncytial virus (RSV) bronchiolitis. Which of the following interventions should be included in the child's care? (Select all that apply.) a. Place in a mist tent. b. Administer antibiotics. c. Administer cough syrup. d. Encourage to drink 8 ounces of formula every 4 hours. e. Cluster care to encourage adequate rest. f. Place on noninvasive oxygen monitoring.

d. Encourage to drink 8 ounces of formula every 4 hours. e. Cluster care to encourage adequate rest. f. Place on noninvasive oxygen monitoring. Rationale: Hydration is important in children with RSV bronchiolitis to loosen secretions and prevent shock. Clustering of care promotes periods of rest. The use of noninvasive oxygen monitoring is recommended

The parents of a child with fragile X syndrome want to have another baby. They tell the nurse that they worry another child might be similarly affected. What is the most appropriate nursing action? a. Reassure them that the syndrome is not inherited. b. Assess for family history of the syndrome. c. Recommend that they do not have another child. d. Explain that prenatal diagnosis of the syndrome is now available.

d. Explain that prenatal diagnosis of the syndrome is now available.

What is an appropriate nursing intervention when caring for an infant with an upper respiratory tract infection and elevated temperature? a. Give tepid water baths to reduce fever. b. Encourage food intake to maintain caloric needs. c. Have child wear heavy clothing to prevent chilling. d. Give small amounts of favorite fluids frequently to prevent dehydration.

d. Give small amounts of favorite fluids frequently to prevent dehydration Rationale: Preventing dehydration by small frequent feedings is an important intervention in the febrile child.

What is one of the major physical characteristics of the child with Down syndrome? a. Excessive height b. Spots on the palms c. Inflexibility of the joints d. Hypotonic musculature

d. Hypotonic musculature Rationale: Hypotonic musculature is one of the major characteristics.

The parents of a child with cerebral palsy ask the nurse if any drugs can decrease their child's spasticity. The nurse's response should be based on which of the following? a. Anticonvulsant medications are sometimes useful for controlling spasticity. b. Medications that would be useful in reducing spasticity are too toxic for use with children. c. Many different medications can be highly effective in controlling spasticity. d. Implantation of a pump to deliver medication into the intrathecal space to decrease spasticity has recently become available.

d. Implantation of a pump to deliver medication into the intrathecal space to decrease spasticity has recently become available. Rationale: Baclofen, given intrathecally, is best suited for children with severe spasticity that interferes with activities of daily living and ambulation.

The mother of a toddler yells to the nurse, "Help! He is choking to death on his food." The nurse determines that lifesaving measures are necessary based on which of the following? a. Gagging b. Coughing c. Pulse over 100 beats/min d. Inability to speak

d. Inability to speak Rationale: The inability to speak is indicative of a foreign-body airway obstruction of the larynx. Abdominal thrusts are needed for treatment of the choking child.

Which of the following statements is characteristic of AOM? a. The etiology is unknown. b. Permanent hearing loss often results. c. It can be treated by intramuscular (IM) antibiotics. d. It is treated with a broad range of antibiotics.

d. It is treated with a broad range of antibiotics Rationale: Historically AOM has been treated with a range of antibiotics. However, new research shows that antibiotics do not improve outcomes in children with uncomplicated AOM.

The nurse is assessing a child who was just admitted to the hospital for observation after a head injury. Which of the following is the most essential part of the nursing assessment to detect early signs of a worsening condition? a. Posturing b. Vital signs c. Focal neurologic signs d. Level of consciousness

d. Level of consciousness Rationale: The most important nursing observation is assessment of the child's level of consciousness. Alterations in consciousness appear earlier in the progression of an injury than do alterations of vital signs or focal neurologic signs.

What is important when caring for a child with myelomeningocele in the preoperative stage? a. Place the child on one side to decrease pressure on the spinal cord. b. Apply a heat lamp to facilitate drying and toughening of the sac. c. Keep the skin clean and dry to prevent irritation from diarrheal stools. d. Measure the head circumference and examine the fontanels for signs that might indicate developing hydrocephalus.

d. Measure the head circumference and examine the fontanels for signs that might indicate developing hydrocephalus.

Cystic fibrosis may affect one system or multiple systems of the body. What is the primary factor responsible for possible multiple clinical manifestations? a. Atrophic changes in the mucosal wall of the intestines b. Hypoactivity of the autonomic nervous system c. Hyperactivity of the apocrine glands d. Mechanical obstruction caused by increased viscosity of exocrine gland secretions

d. Mechanical obstruction caused by increased viscosity of exocrine gland secretions

Which of the following refers to a hernial protrusion of a saclike cyst of meninges, spinal fluid, and a portion of the spinal cord with its nerves through a defect in the vertebral column? a. Rachischisis b. Encephalocele c. Meningocele d. Myelomeningocele

d. Myelomeningocele Rationale: A myelomeningocele has a visible defect with an external saclike protrusion, containing meninges, spinal fluid, and nerves.

What clinical manifestation would the nurse expect when a pneumothorax occurs in a neonate who is undergoing mechanical ventilation? a. Barrel chest b. Wheezing c. Thermal instability d. Nasal flaring and retractions

d. Nasal flaring and retractions

Pancreatic enzymes are administered to the child with CF. Nursing considerations should include which of the following? a. Do not administer pancreatic enzymes if child is receiving antibiotics. b. Decrease dose of pancreatic enzymes if child is having frequent, bulky stools. c. Administer pancreatic enzymes between meals if at all possible. d. Pancreatic enzymes can be swallowed whole or sprinkled on a small amount of food taken at the beginning of a meal.

d. Pancreatic enzymes can be swallowed whole or sprinkled on a small amount of food taken at the beginning of a meal. Rationale: Enzymes may be administered in a small amount of cereal or fruit at the beginning of a meal or swallowed whole.

The nurse is caring for a child with severe head trauma after a car accident. Which of the following is an ominous sign that often precedes death? a. Papilledema b. Delirium c. Doll's head maneuver d. Periodic and irregular breathing

d. Periodic and irregular breathing Rationale: Periodic or irregular breathing is an ominous sign of brainstem (especially medullary) dysfunction that often precedes complete apnea.

Which of the following drugs is usually given first in the emergency treatment of an acute, severe asthma episode in a young child? a. Ephedrine b. Theophylline c. Aminophylline d. Short-acting β2-agonists

d. Short-acting β2-agonists Rationale: Short-acting β2 agonists are the first treatment in an acute asthma exacerbation.

The parent of an infant with nasopharyngitis should be instructed to notify the health professional if the infant: a. becomes fussy. b. has a cough. c. has a fever over 99° F. d. shows signs of an earache.

d. Shows signs of an earache. Rationale: If an infant with nasopharyngitis shows signs of an earache, it may mean a secondary bacterial infection is present and should be referred to a practitioner for evaluation.

Which of the following phrases describes a characteristic of most neonatal seizures? a. Generalized seizure b. Tonic-clonic seizure c. Well-organized seizure d. Subtle and barely discernible seizure

d. Subtle and barely discernible seizure

The nurse is interviewing the parents of a 4-month-old infant brought to the hospital emergency department. The infant is dead on arrival, and no attempt at resuscitation is made. The parents state that the baby was found in the crib with a blanket over the head, lying face down in bloody fluid from the nose and mouth. The parents indicate no problems when the infant was placed in the crib asleep. Which of the following causes of death does the nurse suspect? a. Suffocation b. Child abuse c. Infantile apnea d. Sudden infant death syndrome (SIDS)

d. Sudden infant death syndrome (SIDS)

CF is suspected in a toddler. Which of the following tests is essential in establishing this diagnosis? a. Bronchoscopy b. Serum calcium c. Urine creatinine d. Sweat chloride test

d. Sweat chloride test Rationale: A sweat chloride test result greater than 60 mEq/L is diagnostic of CF.

A woman who is 6 weeks pregnant tells the nurse that she is worried her baby might have spina bifida because of a family history. What should the nurse's response be based on? a. There is no genetic basis for the defect. b. Prenatal detection is not possible yet. c. Chromosomal studies done on amniotic fluid can diagnose the defect prenatally. d. The concentration of α-fetoprotein in amniotic fluid can potentially indicate the presence of the defect prenatally.

d. The concentration of α-fetoprotein in amniotic fluid can potentially indicate the presence of the defect prenatally.

Many of the physical characteristics of Down syndrome present nursing problems. Care of the child should include which of the following? a. Delay feeding solid foods until the tongue thrust has stopped. b. Modify diet as necessary to minimize the diarrhea that often occurs. c. Provide calories appropriate to child's age. d. Use a cool-mist vaporizer to keep mucous membranes moist.

d. Use a cool-mist vaporizer to keep mucous membranes moist. Rationale: The constant stuffy nose forces the child to breathe by mouth, drying the mucous membranes and increasing the susceptibility to upper respiratory tract infections. A cool-mist vaporizer will keep the mucous membranes moist and liquefy secretions.

The mother of a 1-month-old infant tells the nurse she worries that her baby will get meningitis like her oldest son did when he was an infant. The nurse should base her response on which of the following? a. Meningitis rarely occurs during infancy. b. Often a genetic predisposition to meningitis is found. c. Vaccination to prevent all types of meningitis is now available. d. Vaccination to prevent Haemophilus influenzae type b meningitis has decreased the frequency of this disease in children.

d. Vaccination to prevent Haemophilus influenzae type b meningitis has decreased the frequency of this disease in children. Rationale: H. influenzae type b meningitis has been virtually eradicated in areas of the world where the vaccine is administered routinely.

It is important that a child with acute streptococcal pharyngitis be treated with antibiotics to prevent a. otitis media. b. diabetes insipidus. c. nephrotic syndrome. d. acute rheumatic fever

d. acute rheumatic fever

An 8-year-old has been diagnosed with moderate cerebral palsy (CP). The child recently began participation in a regular classroom for part of the day. The child's mother asks the school nurse about joining the after-school Scout troop. The nurse's response should be based on knowledge that a. most activities such as Scouts cannot be adapted for children with CP. b. after-school activities usually result in extreme fatigue for children with CP. c. trying to participate in activities such as Scouts leads to lowered self-esteem in children with CP. d. after-school activities often provide children with CP with opportunities for socialization and recreation.

d. after-school activities often provide children with CP with opportunities for socialization and recreation.

One of the goals for children with asthma is to prevent respiratory tract infection because infections a. lessen effectiveness of medications. b. encourage exercise-induced asthma. c. increase sensitivity to allergens. d. can trigger an episode or aggravate asthmatic state.

d. can trigger an episode or aggravate asthmatic state.

The most appropriate nursing intervention when caring for a child experiencing a seizure is to a. restrain the child when a seizure occurs to prevent bodily harm. b. place a padded tongue between the teeth if they become clenched. c. suction the child during the seizure to prevent aspiration. d. described and document the seizure activity observed.

d. described and document the seizure activity observed.

The school nurse is called to the cafeteria because a child "has eaten something he is allergic to." The child is in severe respiratory distress. The first action by the nurse is to a. determine what the child has eaten. b. administer diphenhydramine (Benadryl) PO stat. c. move the child to the nurse's office or hallway. d. have someone call for an ambulance and paramedic rescue squad or 9-1-1.

d. have someone call for an ambulance and paramedic rescue squad or 9-1-1.

A child is unconscious after a motor vehicle accident. The watery discharge from the nose tests positive for glucose. The nurse should recognize that this suggests: a. diabetic coma. b. brainstem injury. c. upper respiratory tract infection. d. leaking of cerebrospinal fluid (CSF).

d. leaking of cerebrospinal fluid (CSF). Rationale: Watery discharge from the nose that is positive for glucose suggests leaking of CSF from a skull fracture.

The nurse is instructing a group of parents about head injuries in children. The nurse should explain that infants are particularly vulnerable to acceleration-deceleration head injuries because the a. anterior fontanel is not yet closed. b. nervous tissue is not well developed. c. scalp of head has extensive vascularity. d. musculoskeletal support of head is insufficient.

d. musculoskeletal support of head is insufficient.

An adolescent gets hit in the eye during a fight. The school nurse, using a flashlight, notes the presence of gross hyphema (hemorrhage into anterior chamber). The nurse should: a. apply a Fox shield. b. instruct the adolescent to apply ice for 24 hours. c. have adolescent rest with eye closed and ice applied. d. notify parents that adolescent needs to see an ophthalmologist.

d. notify parents that adolescent needs to see an ophthalmologist. Rationale: The parents should be notified that the adolescent needs to see an ophthalmologist as soon as possible.

A 4-year-old child is brought to the emergency department. The child has a "froglike" croaking sound on inspiration, is agitated, and is drooling. The child insists on sitting upright. The priority action by the nurse is to a. examine the child's oropharynx and report the assessment to the healthcare provider. b. make the child lie down and rest quietly. c. auscultate the child's lungs and make preparations for placement in a mist tent. d. notify the healthcare provider immediately and be prepared to assist with a tracheostomy or intubation.

d. notify the healthcare provider immediately and be prepared to assist with a tracheostomy or intubation

A 5-year-old has bilateral eye patches in place after surgery one day earlier. Today, the child can be out of bed. The most appropriate nursing intervention is to a. reassure the child and allow the parents to stay b. allow the child to assist in self-feeding c. speak to the child when entering the room d. orient the child to the immediate surroundings

d. orient the child to the immediate surroundings

Skin testing for tuberculosis (TB) (the Mantoux test) is recommended: a. every year for all children older than 2 years. b. every year for all children older than 10 years. c. every 2 years for all children starting at age 1 year. d. periodically for children who reside in high-prevalence regions.

d. periodically for children who reside in high-prevalence regions Rationale: Children who reside in high-prevalence regions for TB should be tested every 2 to 3 years.

Cerebral palsy (CP) may result from a variety of causes. It is now known that the most common cause of CP is a. birth asphyxia b. neonatal diseases c. cerebral trauma d. prenatal brain abnormalities

d. prenatal brain abnormalities

The major goal of therapy for children with cerebral palsy (CP) is a. reversing degenerative processes that have occurred. b. curing the underlying defect causing the disorder. c. preventing spread to individuals in close contact with the children. d. recognizing the disorder early and promoting optimal development.

d. recognizing the disorder early and promoting optimal development.

A 2-week-old infant with Down syndrome is being seen in the clinic. The mother tells the nurse that the infant is difficult to hold. "The baby is like a rag doll and doesn't cuddle up to me like my other babies did." The nurse interprets the infant's behavior as a a. sign of maternal deprivation. b. sign of detachment and rejection. c. sign of autism associated with Down syndrome. d. result of the physical characteristics of Down syndrome.

d. result of the physical characteristics of Down syndrome.

A humidified atmosphere is recommended for a young child with an upper respiratory tract infection because it a. liquefies secretions. b. improves oxygenation. c. promotes ventilation. d. soothes inflamed mucous membrane

d. soothes inflamed mucous membrane

A neural tube defect that is not visible externally in the lumbosacral area would be called a. meningocele. b. myelomeningocele. c. spina bifida cystica. d. spina bifida occulta.

d. spina bifida occulta.

The diagnosis of intellectual disability is based on the presence of a. intelligence quotient (IQ) of 75 or less. b. IQ of 70 or less. c. subaverage intellectual functioning, deficits in adaptive skills, and onset at any age. d. subaverage intellectual functioning, deficits in adaptive skills, and onset before 18 years of age.

d. subaverage intellectual functioning, deficits in adaptive skills, and onset before 18 years of age.

The genetic testing of a child with Down syndrome showed that the disorder was caused by chromosomal translocation. The parents ask about further genetic testing. Based on the nurse's knowledge of genetics, the most appropriate recommendation is a.no further genetic testing of the family is indicated. b. the child should be retested to confirm the diagnosis of Down syndrome. c. the mother should be tested if she is over age 35. d. the parents can be tested, since it might be hereditary.

d. the parents can be tested, since it might be hereditary.

Children taking phenobarbital and/or phenytoin may experience a deficiency of: a. calcium. b. vitamin C. c. fat-soluble vitamins. d. vitamin D and folic acid.

d. vitamin D and folic acid. Rationale: Deficiencies of vitamin D and folic acid have been reported in children taking phenobarbital and phenytoin.


Set pelajaran terkait

Quiz -- Chapter 2 -- Business 101

View Set

Chapters 9, 10, and 16 - Abnormal Psychology

View Set

Chapter 5 Understanding Children from Birth to Age Two

View Set

AU 60 - Terms - Commercial Underwriting Principles

View Set

Marketing Research for Managers: Final Exam Study Guide

View Set

Cultural Pluralism - Midterm Review

View Set